Этого треда уже нет.
Это копия, сохраненная 7 июня 2021 года.

Скачать тред: только с превью, с превью и прикрепленными файлами.
Второй вариант может долго скачиваться. Файлы будут только в живых или недавно утонувших тредах. Подробнее

Если вам полезен архив М.Двача, пожертвуйте на оплату сервера.
Тред изучения математики #3 # OP 823017 В конец треда | Веб
Поскольку математического образования в России нет, изучать математику нужно самостоятельно. В этом треде можно координироваться тем, кто получает математическое образование. Тут будет постепенно обновляющийся список литературы (русско- и англоязычной) и набор ссылок.

Ориентир - старая программа Вербицкого:
http://imperium.lenin.ru/~verbit/MATH/programma.html

Архив матфака ВШЭ: https://math.hse.ru/archive
Архив НМУ: https://ium.mccme.ru/idx.html
Новая программа Вербицкого: http://verbit.ru/Job/HSE/Curriculum/all.txt

В предыдущем треде (>>813706 (OP)) некоторые пасты о теоркате >>814264 → >>814271 → и несколько провинциальных мехматов.
2 823018
Линейная алебра 1
Мальцев. Основы линейной алгебры, первое издание
Кострикин. Введение в алгебру, т2
Шафаревич, Ремизов. Линейная алгебра и геометрия
Ефимов, Розендорн. Линейная алгебра и многомерная геометрия
Акивис, Гольдберг. Тензорное исчисление
Гельфанд. Лекции по линейной алгебре
Халмош. Конечномерные векторные пространства
Дьедонне. Линейная алгебра и элементарная геометрия

Линейная алгебра 2
Вербицкий, Каледин. Тривиум. Алгебра
Кострикин, Манин. Линейная алгебра и геометрия
Sergei Winitzki. Coordinate free linear algebra via exterior product
Sheldon Axler. Linear Algebra Done Right
Sergei Treil. Linear Algebra Done Wrong
Прасолов. Задачи и теоремы линейной алгебры
Гантмахер. Теория матриц
Маркус, Минк. Обзор по теории матриц и матричных неравенств

Общая алгебра 1
Шафаревич. Основные понятия алгебры
Chevalley. Fundamental concepts of Algebra
Кострикин. Введение в алгебру, т1
Курош. Курс высшей алгебры

Общая алгебра 2
Кострикин. Введение в алгебру, т3
Городенцев. Алгебра 1
Городенцев. Алгебра 2
Постников. Теория Галуа
Артин. Теория Галуа
Зуланке, Онищик. Алгебра и геометрия, тт1 и 2
Винберг. Курс алгебры
Ван дер Варден. Алгебра
Ленг. Алгебра
MacLane. Birkhoff. Algebra
Dummit, Foote. Abstract algebra

Дискретная математика 1
Стенли. Перечислительная комбинаторика
Ландо. Введение в дискретную математику
Холл. Комбинаторика
Оре. Теория графов

Теория представлений 1
Этингоф. Введение в теорию представлений
Фултон, Харрис. Теория представлений. Начальный курс.
Желобенко. Введение в теорию представлений
Фултон. Таблицы Юнга и их приложения к теории представлений и геометрии

Коммутативная алгебра
Атья, Макдональд. Введение в коммутативную алгебру
David Eisenbud. Commutative algebra, with a view toward algebraic geometry
Бурбаки. Коммутативная алгебра
The CRing Project

Теория категорий 1
Francis Borceux. Handbook of categorical algebra т1
Awodey. Category theory
Pareigis. Categories and Functors
Букур, Деляну. Введение в теорию категорий и функторов
Цаленко, Шульгейфер. Основы теории категорий
Francis Borceux. Handbook of categorical algebra т1
Mac Lane. Categories for the Working Mathematician

Теория категорий 2
Francis Borceux. Handbook of categorical algebra тт2-3
Freyd, Scedrov. Categories, allegories
Adamek, Herrlich, Strecker. The joy of cats

Теория топосов
Голдблатт. Топосы. Категорный анализ логики
Peter T. Johnstone. Sketches of an Elephant
Джонстон. Теория топосов

Гомологическая алгебра
Гротендик. О некоторых вопросах гомологической алгебры
Манин. Гомологическая алгебра
Картан, Эйленберг. Гомологическая алгебра
Маклейн. Гомология
Marcus. An Introduction to Homological Algebra
Rotman. An introduction to homological algebra
Бурбаки. Гомологическая алгебра

Общая топология
Вербицкий. Начальный курс топологии в листочках
Энгелькинг. Общая топология
Келли. Общая топология

Алгебраическая топология
Хатчер. Алгебраическая топология
Фукс, Фоменко. Курс гомотопической топологии
Спеньер. Алгебраическая топология
Свитцер. Алгебраическая топология
Дольд. Лекции по алгебраической топологии
Годеман. Алгебраическая топология и теория пучков
Хилтон, Уайли. Теория гомологий. Введение в алгебраическую топологию
Стинрод. Эйленберг. Основания алгебраической топологии

Геометрия 1
Рыжков. Лекции по аналитической геометрии
Вербицкий, Каледин. Тривиум. Геометрия
Сосинский. Геометрии
Берже. Геометрия
Понарин. Аффинная и проективная геометрия
Бэр. Линейная алгебра и проективная геометрия
Вольберг. Основные идеи проективной геометрии

Логика 1
Манин. Доказуемое и недоказуемое
Колмогоров, Драгалин. Математическая логика (Введение и Дополнительные главы)
Клини. Введение в метаматематику
Клини. Математическая логика
Шенфилд. Математическая логика
Мендельсон. Введение в математическую логику
Френкель, Бар-Хиллел. Основания теории множеств

Теория множеств 1
Виленкин. Рассказы о множествах
Хаусдорф. Теория множеств
Архангельский. Канторовская теория множеств
Александров. Введение в теорию множеств и общую топологию
Ciesielski. Set theory for the working mathematician
Куратовский, Мостовский. Теория множеств

Теория множеств 2
Йех. Теория множеств и метод форсинга
Коэн. Теория множеств и континуум-гипотеза
Вавилов. Не совсем наивная теория множеств
Jech. Set theory the third millennium edition
Барвайс. Справочная книга по математической логике
Kanamori. The higher infinite
Drake. An Introduction to Large Cardinals
Comfort, Negrepontis. The Theory of UltraFilters

Теория множеств 3
Kanamori, Foreman. Handbook of Set Theory
2 823018
Линейная алебра 1
Мальцев. Основы линейной алгебры, первое издание
Кострикин. Введение в алгебру, т2
Шафаревич, Ремизов. Линейная алгебра и геометрия
Ефимов, Розендорн. Линейная алгебра и многомерная геометрия
Акивис, Гольдберг. Тензорное исчисление
Гельфанд. Лекции по линейной алгебре
Халмош. Конечномерные векторные пространства
Дьедонне. Линейная алгебра и элементарная геометрия

Линейная алгебра 2
Вербицкий, Каледин. Тривиум. Алгебра
Кострикин, Манин. Линейная алгебра и геометрия
Sergei Winitzki. Coordinate free linear algebra via exterior product
Sheldon Axler. Linear Algebra Done Right
Sergei Treil. Linear Algebra Done Wrong
Прасолов. Задачи и теоремы линейной алгебры
Гантмахер. Теория матриц
Маркус, Минк. Обзор по теории матриц и матричных неравенств

Общая алгебра 1
Шафаревич. Основные понятия алгебры
Chevalley. Fundamental concepts of Algebra
Кострикин. Введение в алгебру, т1
Курош. Курс высшей алгебры

Общая алгебра 2
Кострикин. Введение в алгебру, т3
Городенцев. Алгебра 1
Городенцев. Алгебра 2
Постников. Теория Галуа
Артин. Теория Галуа
Зуланке, Онищик. Алгебра и геометрия, тт1 и 2
Винберг. Курс алгебры
Ван дер Варден. Алгебра
Ленг. Алгебра
MacLane. Birkhoff. Algebra
Dummit, Foote. Abstract algebra

Дискретная математика 1
Стенли. Перечислительная комбинаторика
Ландо. Введение в дискретную математику
Холл. Комбинаторика
Оре. Теория графов

Теория представлений 1
Этингоф. Введение в теорию представлений
Фултон, Харрис. Теория представлений. Начальный курс.
Желобенко. Введение в теорию представлений
Фултон. Таблицы Юнга и их приложения к теории представлений и геометрии

Коммутативная алгебра
Атья, Макдональд. Введение в коммутативную алгебру
David Eisenbud. Commutative algebra, with a view toward algebraic geometry
Бурбаки. Коммутативная алгебра
The CRing Project

Теория категорий 1
Francis Borceux. Handbook of categorical algebra т1
Awodey. Category theory
Pareigis. Categories and Functors
Букур, Деляну. Введение в теорию категорий и функторов
Цаленко, Шульгейфер. Основы теории категорий
Francis Borceux. Handbook of categorical algebra т1
Mac Lane. Categories for the Working Mathematician

Теория категорий 2
Francis Borceux. Handbook of categorical algebra тт2-3
Freyd, Scedrov. Categories, allegories
Adamek, Herrlich, Strecker. The joy of cats

Теория топосов
Голдблатт. Топосы. Категорный анализ логики
Peter T. Johnstone. Sketches of an Elephant
Джонстон. Теория топосов

Гомологическая алгебра
Гротендик. О некоторых вопросах гомологической алгебры
Манин. Гомологическая алгебра
Картан, Эйленберг. Гомологическая алгебра
Маклейн. Гомология
Marcus. An Introduction to Homological Algebra
Rotman. An introduction to homological algebra
Бурбаки. Гомологическая алгебра

Общая топология
Вербицкий. Начальный курс топологии в листочках
Энгелькинг. Общая топология
Келли. Общая топология

Алгебраическая топология
Хатчер. Алгебраическая топология
Фукс, Фоменко. Курс гомотопической топологии
Спеньер. Алгебраическая топология
Свитцер. Алгебраическая топология
Дольд. Лекции по алгебраической топологии
Годеман. Алгебраическая топология и теория пучков
Хилтон, Уайли. Теория гомологий. Введение в алгебраическую топологию
Стинрод. Эйленберг. Основания алгебраической топологии

Геометрия 1
Рыжков. Лекции по аналитической геометрии
Вербицкий, Каледин. Тривиум. Геометрия
Сосинский. Геометрии
Берже. Геометрия
Понарин. Аффинная и проективная геометрия
Бэр. Линейная алгебра и проективная геометрия
Вольберг. Основные идеи проективной геометрии

Логика 1
Манин. Доказуемое и недоказуемое
Колмогоров, Драгалин. Математическая логика (Введение и Дополнительные главы)
Клини. Введение в метаматематику
Клини. Математическая логика
Шенфилд. Математическая логика
Мендельсон. Введение в математическую логику
Френкель, Бар-Хиллел. Основания теории множеств

Теория множеств 1
Виленкин. Рассказы о множествах
Хаусдорф. Теория множеств
Архангельский. Канторовская теория множеств
Александров. Введение в теорию множеств и общую топологию
Ciesielski. Set theory for the working mathematician
Куратовский, Мостовский. Теория множеств

Теория множеств 2
Йех. Теория множеств и метод форсинга
Коэн. Теория множеств и континуум-гипотеза
Вавилов. Не совсем наивная теория множеств
Jech. Set theory the third millennium edition
Барвайс. Справочная книга по математической логике
Kanamori. The higher infinite
Drake. An Introduction to Large Cardinals
Comfort, Negrepontis. The Theory of UltraFilters

Теория множеств 3
Kanamori, Foreman. Handbook of Set Theory
3 823036
Для чего все это нужно? Как вообще происходят занятия чистой математикой? Сидит человек наш нерешённой задачей, думает, пробует разные подходы из разных областей математики?
И как вообще новые области появляются...
20101209.gif85 Кб, 540x701
# OP 4 823058
>>23036

>Для чего все это нужно?


См. второй стрип.
5 823069
>>23018
Есть смысл Мальцева по линалу читать? В эдвайс-листе есть, но пояснений не видел. Кострикина-Манина не осилил.
# OP 6 823076
>>23069
Мне весьма нравится, но только первое издание, пикрелейтед.

У книги есть особенности. Во-первых, в первом издании нет определения определителей, а только их использование. Определители нужно уже откуда-то знать.

Во-вторых, символ линейных преобразований записывается справа, в немецком стиле, из-за этого матричная терминология отличается от современной дефолтной на транспонирование.

Второе и последующее издания переделали, чтобы книга соответствовала свежепринятым тогда стандартам советской высшей школы. Стало хуже. Например, геометрическую теорию нормальных форм под влиянием Куроша заменили многочленными лямбда-матрицами ради довольно-таки примитивного подобия теории нормальной формы Смита. Зачем было это делать, непонятно. Ещё и теорию определителей всунули, причём неинтуитивным способом - рекурсивно, аксиоматизировав разложение Лапласа и убив его красивое доказательство.

Для меня отсутствие определения определителя в первом издании - плюс. Я не очень люблю стандартный путь с подстановками и выписыванием слагаемых. А нормальное определение определителя требует внешнюю алгебру и функторные морфизмы. Однако если определители - совсем непонятная штука (т.е. если ты не можешь вручную посчитать определитель матрицы 3 на 3 или выписать разложение определителя по последнему столбцу), то можно взять ну хотя бы книжку "Головина. Линейная алгебра и некоторые приложения" и при необходимости обращаться к первой её главе. В принципе, можно даже книгу этого Куроша взять, про определители в ней доходчиво.

Только всё-таки добавлю дисклеймер. Книга Мальцева - это именно элементарное введение для начинающих. Нужное, чтобы было легче читать более сложные книги типа того же Кострикина-Манина. После Мальцева очень полезно прочитать какой-нибудь нормальный абстрактный учебник, чтобы понять внешнюю алгебру. Подойдёт Городенцев. Очень советую листочки Вербицкого-Каледина, которые тривиум. Ещё можно Sergei Winitzki. Linear algebra via exterior product.
7 823098
>>23018
А можно в этом списке литературы выделить евреев, что бы я не тратил время на проверку биографии автора?
8 823100
>>23098
Уйди.
1A7728EE-F11F-49EE-BE4E-0E15C9DA81FA.jpeg141 Кб, 640x640
9 823101
>>23098
Оставайся!
cd25058d0c3456218216fe7b1459351d.jpg26 Кб, 437x563
# OP 10 823103
>>23101
Нет, серьёзно. Антисемитизм не нужен.
11 823105
>>23103
Почему анти?
12 823106
>>23076
Спасибо большое, оп, приму к сведению.
Ну и еще один вопрос в догонку: про логику (вроде не обсуждали ни разу). Раньше ее изучал только по слайдам Л. Беклемишева, плюс решал задачки из Верещагина-Шеня/Лаврова-Максимовой, а теперь надо бы разобраться поподробнее с теорией. Можешь пояснить за выбор между Колмогоровым-Драгалиным/Мендельсоном/Клини? Первый есть в печатном виде, остальные могу найти. Я вроде книги просмотрел, обозначения достаточно отличаются, подходы тоже, боюсь привыкнуть к чему-нибудь дико архаичному.
13 823112
>>23103

>Нет, серьёзно. Антисемитизм не нужен.


Тыскозал?
14 823113
>>23098
Там только Шафаревич русский.
photo2020-12-3019-00-04.jpg83 Кб, 960x1280
# OP 15 823116
>>23106
Смотря что тебе нужно от логики. Если просто основные понятия, то подойдёт Колмогоров-Драгалин, да. По идее, это двухтомник - Введение в математическую логику и Дополнительные главы математической логики, но его часто издают одним томом. В дополнительных главах излагается ZFC, элементы теории рекурсии и алгоритмов, а заканчивается всё теоремами Гёделя о неполноте и полноте, теоремами Лёба и Россера и ещё чем-то в этом духе. Мне кажется, если не специализироваться на логике, то Колмогоров-Драгалин - хорошее введение.

У Мендельсона в качестве теории множеств NBG вместо ZFC, но отличия в основном технические. Эта книжка посложнее Колмогорова (субъективно), и, кажется, несколько лучше подходит, если есть желание поизучать форсинг.

У Клини на русский переведены две книжки: введение в метаматематику и математическая логика. Вторая - переделанный вариант первой, нужный, чтобы преподавать в вузах. А первая написана ради искусства. В принципе, обе довольно заметно устарели. В "метаматематике" стоит прочитать интро, оно там интересное. Дальше, наверное, всё-таки лучше по Колмогорову.

Единственное, что может несколько оттолкнуть от Колмогорова, - там сразу же используется какая-то наивная теория множеств в метаязыке. Но это не критично и порочным кругом на самом деле не является.

Стандартный курс "дискретной математики", если он зачем-то нужен, ни одна из этих книжек не покрывает. Всякие там полиномы Жегалкина и КНФ с ДНФ и резолюциями можно посмотреть у какого-нибудь Яблонского, если тебе вдруг интересна CS.

>обозначения достаточно отличаются


В логике сейчас нет устоявшегося канона. Какие-то теоретики не мыслят логику в отрыве от моделей. Для учебников такого стиля обязательны булевы функции, оценки и интерпретации, неограниченное использование семантической терминологии, всяких там таблиц истинности и геометрических истолкований предикатов. Есть другие теоретики, которые очень жёстко разделяют синтаксис и семантику (к таким можно, с натяжкой, отнести Клини). Очень характерный пример этого стиля - Бурбаки. У него вообще нет никакой семантики, нет понятий истины и лжи, никаких таблиц булевых функций. Только голый синтаксис и синтаксические преобразования. Алсо, для чтения Бурбаки нужно знать теорию тау- и эпсилон-операторов Гильберта, про которые можно прочитать, собственно, у Гильберта-Аккермана и Гильберта-Бернайса.

Из обозначений важно знать разницу между ⊢ и ⊨. А, скажем, каким символом обозначается импликация (→, ⇒, ⊃, etc) не особенно важно. Кому как привычнее. Обычно всё-таки импликация - это →, символом ⊃ редко пользуются, а стрелка ⇒ несколько чаще импликации обозначает логическое следствие. Но, повторюсь, конкретные значки вообще не важны. Любой человек в теме поймёт всё с полуслова, какой бы ни был выбран набор закорючек.

Канонического набора аксиом логики тоже сейчас нет. Раньше каноном была аксиоматика Гильберта плюс аксиомы формальной арифметики (один вариант приводит Клини, несколько менее отчётливо их формулирует Колмогоров). Сейчас люди используют совсем очень разные аксиоматики, и часто с меньшим количеством аксиом. В том числе смешивают семантику с синтаксисом, так иногда делает как раз Беклемишев. В принципе, особого смысла анализировать разницу между наборами аксиом я не вижу. Теоремы важнее. Не только чисто логические, но и теоремы из теории моделей - теорема Гёделя о компактности, теоремы Лёвенгейма-Сколема о повышении и понижении мощностей, другие такие вещи.

>дико архаичному


Если интересна реальная архаика, то ради лулзов можно посмотреть книжку Гетмановой по логике для гуманитариев. Там про логику, какой она была до появления математической логики. Например, логические квадраты Аристотеля описаны. Только лучше взять новое издание, из которого весь ритуальный марксизм выкинули. Впрочем, эта архаика тоже относительная. Некоторые люди, в основном среди философов, до сих пор не вылезают из XIX века, не принимают матлогику и всерьёз обсуждают все эти модусы. https://philosophy.stackexchange.com/questions/17935/how-do-i-use-the-barbara-celarent-etc-mnemonic - типа такого.

А вообще-то все учебники по логике и множествам на русском языке устарели. Например, одним из основных инструментов исследования в теории множеств и вообще в логике является форсинг. Но ни в одном из используемых учебников форсинг не описан даже на зачаточном уровне. Тем более нет учебника со внятным описанием форсинга. Поэтому я всё-таки думаю, что лучше прочитать Колмогорова-Драгалина, чтобы быстро вкатиться в тему, а потом, если захочется, выучиться серьёзной логике уже по какой-нибудь нормальной современной англоязычной книжке.

Впрочем, на русском есть четырёхтомная Справочная книга под редакцией Барвайса. В ней есть и изложение логики, и некоторый рассказ о форсинге, и много всяких других интересных вещей. Использовать ей в качестве основного учебника, наверное, тяжко, но вот открыть и посмотреть я таки рекомендую. В первом томе первая глава поясняет за логику первого порядка, во втором томе первая глава ZFC, четвёртая глава - форсинг; и там ещё есть про форсинг Мартина и комбинаторные варианты форсинга. В четвёртом томе шестая глава - годное введение в топосы.
photo2020-12-3019-00-04.jpg83 Кб, 960x1280
# OP 15 823116
>>23106
Смотря что тебе нужно от логики. Если просто основные понятия, то подойдёт Колмогоров-Драгалин, да. По идее, это двухтомник - Введение в математическую логику и Дополнительные главы математической логики, но его часто издают одним томом. В дополнительных главах излагается ZFC, элементы теории рекурсии и алгоритмов, а заканчивается всё теоремами Гёделя о неполноте и полноте, теоремами Лёба и Россера и ещё чем-то в этом духе. Мне кажется, если не специализироваться на логике, то Колмогоров-Драгалин - хорошее введение.

У Мендельсона в качестве теории множеств NBG вместо ZFC, но отличия в основном технические. Эта книжка посложнее Колмогорова (субъективно), и, кажется, несколько лучше подходит, если есть желание поизучать форсинг.

У Клини на русский переведены две книжки: введение в метаматематику и математическая логика. Вторая - переделанный вариант первой, нужный, чтобы преподавать в вузах. А первая написана ради искусства. В принципе, обе довольно заметно устарели. В "метаматематике" стоит прочитать интро, оно там интересное. Дальше, наверное, всё-таки лучше по Колмогорову.

Единственное, что может несколько оттолкнуть от Колмогорова, - там сразу же используется какая-то наивная теория множеств в метаязыке. Но это не критично и порочным кругом на самом деле не является.

Стандартный курс "дискретной математики", если он зачем-то нужен, ни одна из этих книжек не покрывает. Всякие там полиномы Жегалкина и КНФ с ДНФ и резолюциями можно посмотреть у какого-нибудь Яблонского, если тебе вдруг интересна CS.

>обозначения достаточно отличаются


В логике сейчас нет устоявшегося канона. Какие-то теоретики не мыслят логику в отрыве от моделей. Для учебников такого стиля обязательны булевы функции, оценки и интерпретации, неограниченное использование семантической терминологии, всяких там таблиц истинности и геометрических истолкований предикатов. Есть другие теоретики, которые очень жёстко разделяют синтаксис и семантику (к таким можно, с натяжкой, отнести Клини). Очень характерный пример этого стиля - Бурбаки. У него вообще нет никакой семантики, нет понятий истины и лжи, никаких таблиц булевых функций. Только голый синтаксис и синтаксические преобразования. Алсо, для чтения Бурбаки нужно знать теорию тау- и эпсилон-операторов Гильберта, про которые можно прочитать, собственно, у Гильберта-Аккермана и Гильберта-Бернайса.

Из обозначений важно знать разницу между ⊢ и ⊨. А, скажем, каким символом обозначается импликация (→, ⇒, ⊃, etc) не особенно важно. Кому как привычнее. Обычно всё-таки импликация - это →, символом ⊃ редко пользуются, а стрелка ⇒ несколько чаще импликации обозначает логическое следствие. Но, повторюсь, конкретные значки вообще не важны. Любой человек в теме поймёт всё с полуслова, какой бы ни был выбран набор закорючек.

Канонического набора аксиом логики тоже сейчас нет. Раньше каноном была аксиоматика Гильберта плюс аксиомы формальной арифметики (один вариант приводит Клини, несколько менее отчётливо их формулирует Колмогоров). Сейчас люди используют совсем очень разные аксиоматики, и часто с меньшим количеством аксиом. В том числе смешивают семантику с синтаксисом, так иногда делает как раз Беклемишев. В принципе, особого смысла анализировать разницу между наборами аксиом я не вижу. Теоремы важнее. Не только чисто логические, но и теоремы из теории моделей - теорема Гёделя о компактности, теоремы Лёвенгейма-Сколема о повышении и понижении мощностей, другие такие вещи.

>дико архаичному


Если интересна реальная архаика, то ради лулзов можно посмотреть книжку Гетмановой по логике для гуманитариев. Там про логику, какой она была до появления математической логики. Например, логические квадраты Аристотеля описаны. Только лучше взять новое издание, из которого весь ритуальный марксизм выкинули. Впрочем, эта архаика тоже относительная. Некоторые люди, в основном среди философов, до сих пор не вылезают из XIX века, не принимают матлогику и всерьёз обсуждают все эти модусы. https://philosophy.stackexchange.com/questions/17935/how-do-i-use-the-barbara-celarent-etc-mnemonic - типа такого.

А вообще-то все учебники по логике и множествам на русском языке устарели. Например, одним из основных инструментов исследования в теории множеств и вообще в логике является форсинг. Но ни в одном из используемых учебников форсинг не описан даже на зачаточном уровне. Тем более нет учебника со внятным описанием форсинга. Поэтому я всё-таки думаю, что лучше прочитать Колмогорова-Драгалина, чтобы быстро вкатиться в тему, а потом, если захочется, выучиться серьёзной логике уже по какой-нибудь нормальной современной англоязычной книжке.

Впрочем, на русском есть четырёхтомная Справочная книга под редакцией Барвайса. В ней есть и изложение логики, и некоторый рассказ о форсинге, и много всяких других интересных вещей. Использовать ей в качестве основного учебника, наверное, тяжко, но вот открыть и посмотреть я таки рекомендую. В первом томе первая глава поясняет за логику первого порядка, во втором томе первая глава ZFC, четвёртая глава - форсинг; и там ещё есть про форсинг Мартина и комбинаторные варианты форсинга. В четвёртом томе шестая глава - годное введение в топосы.
16 823120
>>23113
Тогда нужно предупреждать людей заранее
bell-curves.png105 Кб, 576x877
17 823124
>>23098

>А можно в этом списке литературы выделить евреев, что бы я не тратил время на проверку биографии автора?



Проебешь 90% матана
18 823127
>>23124
Какие молодцы евреи. Это исследование убедило меня в том, что в России нужно как можно больше евреев. И желательно, чтобы они контролировали все самые важные социальные институты типа государственного аппарата, университетов, телевидения и тд.
19 823187
Тут есть те, кто практиковал максимально интенсивное освоение больших объемов информации за кратчайшие сроки?
Мне нужно будет через месяц за 4 дня заботать семестровый курс аналитической геометрии. Знаю, что это неправильный подход, но иного выхода у меня нет.
Это возможно? Если возможно, то останутся ли у меня какие-нибудь знания после сдачи экзамена?
Планирую решать задачки из предыдущих экзаменов, а также прочитать введение в алгебру Кострикина, т1. Там как раз материал первой половины учебника будет в экзамене.
20 823189
>>23187
Возможно конечно. Знания улетучатся прям на выходе из аудитории, я серьезно.
sage 21 823191
>>23124

>(((айкью тесты)))

# OP 23 823238
>>23187

> за 4 дня заботать семестровый курс аналитической геометрии.


Конкретно вот это сделать скорее реально, чем нет. Там не очень много контента, если не запоминать выкладки дословно. Но почему бы не начать раньше?
24 823243
Посоветуйте задачники по матану и линалу чтоб подготовиться к Я - профессионал и другим студенческим контестам по математике. Главное чтобы было много авторских решений задач, а не просто какая-то сухая выкладка по теории и полтора примера на каждую тему как в ефимове-демидовиче.
# OP 25 823246
>>23243
Кстати, присоединяюсь к реквесту.
26 823258
https://www.edx.org/course/how-to-learn-math-for-students-2

Если кому интересно. Бесплатный курс, где рассказывается про типичные затупы в изучении математики.

Идея первая. Есть два типа mindset. Growth mindset and fixed mindset.
Люди с первым не сдаются перед трудностями, а ошибки их только подталкивают пробовать новые пути решения.

Вторая идея. Математикой лучше заниматься в группах. Там приводиться исследование про студентов. Одни занимались в группе и были успешными. Вторые занимались в одиночку и вылетели из топового универа США.
Когда вторые начали заниматься в группе, то у них получилось дорасти до учёных математиков и всё такое.

Есть ли желающие, которые хотят вместе изучать математику? Можно организоваться и помогать друг другу. (И няшиться под пледиком, если вы куны)

Третья идея.
Математика это не про скорость решения задач. Это про спокойное и глубокое изучение предмета.
Там ламповые цитаты математика, который считал в школе, что он идиот, ТК там требовалась быстрая скорость решения задач.

Если кого заинтересовало, то отпишитесь в тредик. Я никогда не организовывал учебные группы, но хочу попробовать. От вас только одно требование - будьте няшками)

Я думаю, что можно организовать общение в дискорде, но нам потребуется какая-то система типо общей доски для написания. Есть какие-то идеи?
27 823259
>>23258

>Одни занимались в группе и были успешными. Вторые занимались в одиночку и вылетели из топового универа США.


Я сознательно выбрал второй путь и вылетел из топового универа РФ. Потом поступил обратно, опять начал заниматься в одиночку и вылетел из вуза ещё раз. Я ебал в рот групповые занятия.
28 823260
>>23259
Аутяры-омегоиды никому не нужны. Это только в фильмах их представляют в роли каких-то гениев и созидателей, в реальной жизни социобляди находятся на верхушке
29 823261
>>23259
Честно говоря, то сочувствую.

Я просто понимаю, что способность работать с незнакомыми людьми и делать работу эффективно это очень круто. Я бы тоже хотел побыть в одиночку, но в реале одиночки добиваются успеха настолько редко, что это немного дебильная тактика.

Я вообще предлагаю просто заниматься и обсуждать какие-то вопросы, которые могли быть поняты кем-то быстрее других.

А почему вылетел? Бухал? Забивал на учебу из депрессии?
30 823303
>>23258
Пиши в лс, готов к совместным занятиям.
31 823345
Включаю вангу: в групповых занятиях уровня 2ч преиспеет тот кто занимается с хорошим репетитором. Остальные потратят время на рофлы, мемасы и тд, в итоге проебутся во всём.
Люди разные, я учился с социоблядями, которые сбивались в группы при том брали они к себе таких же социоблядей. Тех кто им не нравился всячески игнорировали, вплоть до того что не здоровались. Как и полагается таким людям, своё поведение они считали нормальным.
Но были и одиночки. Один из них вообще обходил по мозгам всю остальную группу вместе взятую. Хотя всегда сидел позади и слушал в пол уха. Я так и не понял, был ли он гением, либо заранее разбирал материал, поэтому на лекциях слушал то что уже знал.
# OP 32 823377
>>23345

>был ли он гением


Гениальные люди бывают, да. Меня регулярные столкновения с ними всегда несколько демотивируют. Но что поделать.
image.png29 Кб, 714x497
а 33 823386
здравствуйте, боги математики. поднимаюсь к вам лишь на секунду, чтобы спросить немного глупый, но важный вопрос.
x+2>0 с какого хуя в ОДЗ? мы его туда вносим только из-за того, что он потом оказывается в аргументе логарифма? если так, то почему мы не вносим это ограничение на x после преобразования? или тут все проще и я проебал какое-то свойство степени. прошу не смеяться надо мной, я новичок в математике
а 34 823387
ух, епта, прочитал я о чем вы тут говорите и насрал в штаны. вы же походу послевузовскую математику обсуждаете, сорян, что зашёл сюда, я новичок в дваче
35 823393
>>23386
В условии твоей задачи какое-то число (x+2) возводят в степень lg2. lg2 - иррациональное число. В школе умеют возводить в иррациональную степень только те числа, которые больше нуля. По-научному говоря: область определения степенной функции есть положительные вещественные числа. По-хорошему в область определения нужно ещё включать ноль, но это не всегда делают.

Короче, посмотри, как в твоих учебниках описана область определения степенной функции.
36 823401
>>23393
Cпасибо
37 823428
>>23018
Для чего имеет смысл изучать теорию множество? У нее есть какое-то прикладное применение или это просто основания?
38 823445
>>23428
Это одна из областей, в которых можно что-нибудь открыть.
39 823531
привет! што скажите насчет napkin от evan chen? и других его книжек?
40 823532
>>23531
Зачем он тебе?
41 823566
>>23531
А что тут говорить? Учиться надобно. Неважно, по napkin ли, по чему-то ещё ли.
42 823575
>>23258
Я заинтересован в совместном изучении. Правда, моя цель - восстановить школьную профильную программу, часть я уже повторил, но сейчас мотивация иссякла. Так что группа была бы кстати.
43 823606
>>23575
Создавайте! Я с вами!!!
44 823615
>>23606
Для тебя вход 500 рублей
45 823646
>>23575
Что можно делать в группе такого, что нельзя сделать итт?
46 823650
>>23646
Постить смишнявки, пиздострадать по залетным тням, сраться, аттеншнхорить в уначе - все, что обычно делают в конфах.
47 823661
>>23650

>Постить смишнявки, пиздострадать по залетным тням, сраться, аттеншнхорить в уначе - все, что обычно делают в конфах.


Нахуй это нужно, если можно просто вкатиться по пастам ОПа оснований-треда в конструктивизм и лампово надрачивать на алгорифмы Маркова?
48 823664
>>23661
вкатился тебе за щеку бро
# OP 49 823667
>>23661

>ОПа оснований-треда


Он же даже ни разу не упоминал ничего из Sketches of an elephant. Он не шарит в конструктивизме, просто какой-то фрик.
D853C983-995F-46C6-8CA8-6F1F78AFD2E4.jpeg73 Кб, 810x613
50 823686
51 823689
>>23686
Спок, зековское быдло.
52 823732
>>23258
Не работает, в итоге один-два человека будут заниматься (они бы и без того занимались), а остальные кекать-пукать.
53 823833
>>23258

Пиши, я в деле!
@manuil
55 823839
>>23833
В каком деле? Что хочешь сделать?
56 823914
>>23833
Я тоже в деле! @soba444ka4pip4pip8
57 823937
>>23914
У нас новые правила, прости, ты не успел. 700 рублей
58 823939
>>23937
Куда скидывать?
59 823970
С чего лучше всего начинать здраво изучать математику? Вот вроде бы и есть какие-то отрывочные знания после сельской школы и пту, но какого-то понимания и видения связей -- нет.
Книги с кружковыми\олимпиадными задачами для пятого класса нормальное начало? Боюсь, что без преподователя я не осилю их, а часть решу не очень рационально.
60 823973
>>23970
Зачем тебе её изучать?
61 823976
>>23973
Эстетически нравится.
62 823992
Скопирую сюда из матх:

Сколько итераций Анону нужно, чтобы отлично знать линал или матан? Заметил, что с каждой итерацией прохождения курса знаю предметы все лучше и лучше, но все равно какая то часть со временем быстро забывается, хотя каждый раз забывается все меньше
63 824003
>>23258
Я за.
gYf~reenletters11ANUSgmh!dailPUNCTUMcd,com
64 824005
>>23992
Много.
photo2021-01-0520-43-44.jpg129 Кб, 1278x958
# OP 65 824095
>>23970
По-видимому, работающий вариант - стандартные университетские учебники для широких народных масс, и стандартные задачники. Обычный Письменный, обычный Проскуряков. Они написаны криво-косо, но после них заметно легче читать какие-то интересные книги. Я знаю людей, которые так вкатились. (решать задачники от корки до корки, конечно, не нужно).

>Книги с кружковыми\олимпиадными задачами


Не факт, что они полезны - кружки и олимпиады обособлены от мейнстрима, у них свой мир. Но почему бы и нет.
66 824099
>>23258
Замечательно. Курс - мусорный мешок от тетки, построившей карьеру в рамках сжв-волны и ратующей за гендерное и расовое равенство в школьном классе. Занятия проводят климактерическая белая тетка, жирная лесбуха, ниггер и пара индусов для антуража. Внимание, внимание, математике снова не хватает баб и негров - нужно больше негров и баб, вот тогда заживем.

Саму тетку поймали на невалидности ресерча, а лежащие в основе ресерча концепции (см. mindset) заимствованы у другой тетки, обвиненной в подтасовке данных и невоспроизводимости результатов исследований.

В курсе делят пироги, ищут математику в хип-хопе и жонглируют цитатами жида из гугла, просравшего сотни полимеров на никому не нужный гугл-гласс. И конечно же, рассказывают старые добрые анекдоты про культурные иконы мурриканского плебса - героя мартышкиного труда Эдисона и негра-баскетболиста Майкла Джордана. Внимание, вопрос: какое отношение вся эта залупа имеет к математическому образованию? Подсказка: никакое.

>типичные затупы в изучении математики


Это не типичные затупы - это бабьи выдумки. Типичные затупы давно обсосаны Пойа, Шенфилдом, Фройденталем, Мэйсоном и десятками других настоящих ученых - но они же все белые цисгендерные хуемрази-угнетатели, они не верят в равенство негров и баб, как же про них можно рассказывать нашей либеральной пансексуальной молодежи?

Только зря время потратил. Даже в каком-нибудь новом альбоме Славы КПСС можно найти больше математики, чем в этой хуйне. Дизлайк, отписка.
67 824117
https://interneturok.ru/subject/matematika/class/6

Что думаете для повторения школьной программы? У меня в школе были орущие учителя, я постоянно школу проебывал.
68 824131
>>24095
А Васильев что в этом списке делает? Это ж пиздец сложная книга, на самом деле.
69 824143
>>24131
Да нет, довольно простая.
70 824479
Аноны, а покажите ваши варианты билетов сессии. Она же уже началась или вот-вот начнётся. Естественно, интересуют только математические предметы
71 824481
>>23575
Вот и я хотел бы наверстать школьный уровень. Создали конфу? Напишите @qalp. Или я создам тогда.
72 824482
>>24481
@qalpj
быстрофикс
73 824483
>>24481
Создавай.
image.png392 Кб, 1733x980
74 824500
Аноны, как максимально быстро вкатиться в понимание таких вещей? В универе был просто матан (который я уже скорей всего успешно забыл).
Пробовал гуглить по-отдельности каждую хрень, а там к каждой хрени ещё такой же список прилагается, т.е. гугл тут не поможет и нужна нормальная система обучения (но без оверхеда, нужен самый минимум)
75 824501
>>24500
Статистика - наука, совсем отдельная от математики. В Гарварде это вообще отдельный департамент
76 824502
>>24501

>отдельная


лолчто? используется же мат. язык
я бы сказал статистика является подвидом математики, как дискретка
77 824504
>>24502
Физика тоже активно использует математику. Но не является математикой.
78 824506
>>24504
Была когда-то частью математики.
79 824507
>>24506
Нет, она с самого начала была отдельной наукой. Более того, когда всякие сомнительные персонажи начали пытаться применять математику в натурфилософии, учёные активно возражали.
80 824508
>>24507
https://ru.wikipedia.org/wiki/Натурфилософия
Вот пруф - математика начинается только у Оксфордских Калькуляторов
81 824512
Математические гении, у меня вопрос гораздо более приземлённого рода - с чего начать изучение современного курса математики? Вот никогда вообще не владел математическими навыками, а сейчас когда появилось свободное время и более-менее место в жизни найдено, захотел именно этим овладеть. Чисто для себя. Школьные учебники какие-нибудь начать читать? Или как?
82 824603
>>24512
https://artofproblemsolving.com/store/list/aops-curriculum
Учат с уровня "как правильно складывать и умножать" до уровня международной Олимпиады для школьников 11 класса.

На ютубе есть лекции по первым книгам. Скачать на либгене.
9D34B618-BB3E-4DE2-908C-8247C6BCD7E4.jpeg49 Кб, 362x512
Зорич 83 824609
>>24512
Читай Зорича, там есть ответы на все вопросы и даже больше.
84 824610
НОВИКОВ ЛУТШИЙ УЧЕБНЕК ВСЕ СОГЛАСНЫ?????????????????
85 824618
>>24603
О, добра. Вот это мне и нужно.
86 824638
>>24603
О пасибо
87 824639
>>24603

>artofproblemsolving


Что за они такие?
88 824690
>>24610
Узнали.
photo2020-12-2323-27-03.jpg43 Кб, 817x419
89 824773
Такие дела.
90 824932
>>23243
Ищи вступительные в ШАД. ИМХО, задачи схожие по крайней мере по матану, линалу, теорверу
91 824934
>>23258
Вы что-то за пределами школьной программы обсуждаете? Если да, то и меня добавьте @PaulEdd
92 824939
>>23017 (OP)

>изучать математику нужно самостоятельно


Кому нужно то, пердолик задротный? Очки не давят?
93 824965
>>24932
Там обычные задачи уровня Проскурякова.
94 824973
>>24965
Тебе открытые проблемы подавай?
95 824976
>>24973
Смысл поста в том, что на этих олимпиадах и в шад не какая-то экзотика, а обычные задачи из массовых задачников.
96 824978
>>24976
Да, есть такое. Но бывает проблематично выделить из всего пособия самые сочные задачи
97 824980
>>24976
Проблема в том что большинство задачников говно и по ним невозможно научиться решать задачи без преподавателя.
98 825009
>>24980
Какое "большинство"? На русском их почти нет.
99 825017
>>25009

>Какое "большинство"?


Я ни одного хорошего не видел.

>На русском их почти нет.


Ну что поделать, я родился в России в русской семье, следовательно разговариваю и читаю на русском языке. Приходится довольствоваться тем что есть.
100 825057
>>25017
Некоторое время назад люди, родившиеся в России, разговаривали и читали на французском языке больше, чем на русском, и это считалось нормой.
101 825059
>>25057

>люди, родившиеся в России, разговаривали и читали на французском языке больше, чем на русском


И сколько таких людей было?
102 825060
Не надо заставлять детей учить английский. Пускай, лучше изучают автомат Калашникова. И тогда скоро весь мир заговорит по-русски.
103 825064
>>25059
Довольно много, если не считать крестьян и провинциалов.
104 825077
>>25064
Ловко ты 90% населения исключил. Узнаю дурачка Вадика по его охуительным выводам. После войны 1812 года популярность французского резко упала, а это 200 лет назад было.
105 825080
>>25077
Ну а какой смысл рассматривать крестьянство? Они научных открытий не делают, просто растут.
106 825098
>>25080
Делали и получали дворянские титулы за свои достижения и заработанные собственными руками деньги
107 825101
Не нашел тред теорвера, спрошу тут.
Как найти вероятность P (A | B, C) через P(A | B) и P(A | C)?
Интуиция подсказывает, что можно, но определение условной вероятности с ней почему-то не соглашается.
108 825103
>>25057

>Некоторое время назад люди, родившиеся в России, разговаривали и читали на французском языке больше, чем на русском, и это считалось нормой.


>Некоторое время назад люди, родившиеся в Англии, разговаривали и читали на норманском языке больше, чем на английском, и это считалось нормой.


>Некоторое время назад люди, родившиеся в Америке, разговаривали и читали на немецком языке больше, чем на английском, и это считалось нормой.

109 825110
>>25101
У тебя события независимые?
110 825111
>>25110
Нет, зависимые. Можно даже найти P(B | C)
111 825120
Посоветуйте толковой литературы по терверу. Ширяева стоит штурмовать? Выглядит довольно сложным
112 825121
>>25120
Для чего? Ширяев в основном по случайным процессам
113 825122
>>25120
Пытьев, Шишмарев
"Теория вероятностей, математическая статистика и элементы теории возможностей для физиков"
114 825255
Реквестирую мысль, которую вы долго не могли осознать, но как осознали, то сразу все встало на свои места. Ну, вы понимаете. Я начну надо сдавать егэ
115 825271
сап
посоветуйте что нибудь плиз про историю возникновения и развития математики
# OP 116 825273
>>25271
Юшкевич, История математики с древнейших времен до начала XIX века.
Юшкевич, История математики в Средние века: Математика до эпохи Возрождения. В трёх томах.
Юшкевич, Колмогоров. Математика XIX века. Тоже в трёх томах.
Бурбаки, Очерки по истории математики.

Всё есть в Либгене.
http://gen.lib.rus.ec/

По XX веку нормальной литературы нет, а ведь именно в XX веке математика, по сути, и появилась. Впрочем, есть Прасолов, История математики.
https://sites.google.com/site/prasolovskacatmoiknigi/neizdannoe-v-processe-napisania
117 825275
>>25273
спасибо!
# OP 118 825279
>>25273
*в трёх томах история "до начала XIX века", а средние века всё-таки однотомник.

>>25275
Вообще, Адольф Юшкевич - один из крупнейших историков математики не только в СССР, но и в мире. Он прожил почти девяносто лет, родился в Российской империи, умер в РФ. Со многими историческими фигурами был знаком лично. Достаточно сказать, что он ученик Егорова - создателя московской математической школы. Небезызвестный сборник "Историко-математические исследования" тоже он запилил. Очень жалко, что Юшкевич не стал описывать XX век.

К слову, в сороковых годах успел позаведовать кафедрой высшей математики в Бауманке, перебрался в ИИЕТ из-за гонений на евреев.
119 825291
>>25279

>сороковых годах успел позаведовать кафедрой высшей математики в Бауманке, перебрался в ИИЕТ из-за гонений на евреев.


Какая же бауманка все таки базированная
121 825487
>>25438
Заебись
F6BFB9BC-B777-497A-B360-670B8E85EEC9.jpeg166 Кб, 628x501
122 825491
>>25438
В целом не был. Антисем##### там происходил лишь иногда и только в некоторых местах (когда в некоторых людях просыпался великорусский шовинизм). А так большую часть времени подчинялись любимым Зимбельрманам (которых по традиции было почему-то слишком много в администрации)
123 825493
>>25491

>Антисем#####


Чем тебе не нравятся семитские народы, что ты их цензуришь?
124 825511
>>25493
Тем, что во время революции и в течение первых 20 лет советской власти они уничтожили практически всю русскую интеллигенцию и заняли ее место (заняли буквально, эти обоссаные евреи-революционеры переезжали из местечек в Мск и Спб целыми семьями и занимали квартиры расстрелянных русских). За что в поздние годы правления Сталина и до конца жизни совка им регулярно прилетала ответочка от коренного населения.

Что характерно, даже эта ответочка имела мало отношения к антисемитизму как таковому. В то время в СССР была жесткая разнарядка на многонационалочку (такая же как сейчас на западе), и 100% жидов на кафедре в нее явно не вписывались, потому что по разнарядке на кафедрах должны были быть представлены все национальности - то есть русские, украинцы, грузины, армяне, казахи, якуты и представители остальных братских народов. Поэтому евреев довольно грубо попросили подвинуться с насиженных мест, чтобы освободить место для всех остальных. Евреи на это, разумеется, очень сильно обиделись - причем так обиделись, что жопа у них до сих пор горит. Какую книгу ни возьмешь, какое интервью ни послушаешь, там обязательно какой-нибудь еврей пожалуется, как его при советской власти обижали. Впрочем, для евреев такая постоянная обида на жизнь весьма характерна - они из своей обиженности всегда пытаются извлечь какую-то выгоду. Они даже холокост ухитрились монетизировать. Порода у людей такая, хитрая и расчетливая.

На матфаке, кстати, промывочка на счет "бедных советских евреев" идет огого какая. Там же сплошные пятидесятисемиты с соответствующими национальными корнями.
# OP 125 825557
>>25511
А тебе доводилось бывать на матфаке?
126 825749
Посоветуйте чё читнуть по теории графов, чтоб не очень душно было.
127 825767
>>25749
Уилсон Введение в теорию графов
128 825788
>>25749
Оре, лол.
129 825791
>>25749
Зорич
130 825801
>>25791
Коли, что там не графов.
131 825803
>>25801
Там есть ответы на все вопросы и даже больше
132 825830
>>25803
Есть ответ на вопрос почему ты пидор?
3223.jpg168 Кб, 1200x829
Программа Ленглендса или теория великого объединения математики 133 825861
Господа математики, не могли бы вы рассказать что из себя представляет эта программа и каково ее значение для математики?
# OP 134 825865
>>25861
https://nplus1.ru/material/2018/03/23/langlands - описание языком научпопа.
http://www.mathnet.ru/present4029 - видеообзор на русском, содержательное начинается на 4:05.
https://projecteuclid.org/download/pdf_1/euclid.bams/1183551573 - классическое введение.
135 825871
>>25865
А ты сам лично что думаешь об этой программе?
136 825909
>>25871
Непонятный вопрос. Ну, она существует.
137 826007
Анончики, есть у кого слив Бояршиного по матеше? https://www.youtube.com/watch?v=zIDqlakvse0&list=PLHe0rhp5NjAN9Wd0C_ZpP8WVB75MnmB8v
138 826020
>>26007
Не обкрадывай деда.
139 826030
>>26007
оплатил месяц курса жирного, там нихуя нет. он бездарь.
# OP 140 826032
>>26007
С такими запросами лучше в ЕГЭ-тред. Там шанс получить ответ выше.
141 826033
>>26032
Что за книга и что за функтор Макаки?
nm.png916 Кб, 1382x778
# OP 142 826036
>>26033
Учебник Дика (Tammo tom Dieck) по представлениям.
https://en.wikipedia.org/wiki/Burnside_category
# OP 143 826205
Некоторая демонстрация того, что логика - отдельная наука и должна преподаваться не на факультетах математики, а на факультетах логики.
144 826209
>>26205
Нам лектор по логике говорил, что все логики рано или поздно едут кукухой. Я ему верю.
Оптический-обман.jpg58 Кб, 500x500
# OP 145 826215
>>26209
Почему-то у некоторых людей считается, что сходить с ума - это весело и/или романтично. И что если намекнуть на свою принадлежность к безумию, то это улучшит репутацию в глазах собеседников. Хотя вообще-то любые проблемы с головой - серьёзные, ничего клёвого в них нет. Математикой или другими науками про рассуждения можно заниматься только с нормально работающим мышлением.
# OP 146 826250
147 826259
>>26250
Непрекращающиеся попытки опа байтить на срачи заебли еще с начала второго треда, но сейчас реально какой-то ахуй с трех курсов сатанализа для погромистов. Так победят.
2387586original.jpg140 Кб, 640x1025
# OP 148 826267
>>26259
Тред про математическое образование в российских вузах (во всех трёх с половиной местах, где оно по бумажкам имеется). Если в ИТМО на парах матана рекомендуют читать Энгельса - это не байт, а ценная информация об уровне преподавания математики в ИТМО, ящитаю.
149 826270
>>26250
>>26259
>>26267
Математического образования в ИТМО нет. Углубленный курс математики преподается только на кафедре КТ и пары кафедр оптики, но не более того. На остальных специальностях есть только предмет "Высшая математика", как во многих других технических вузах средней руки.

И то, на КТ все студенты хуй клали на матанализ/функан/алгебру, так как они никому не нужены - в ИТМО не изучают компьютерные технологии, требующие математический аппарат сильно продвинутее школьной математики и самым основам анализа 1 курс (пределы, производные, функции нескольких переменных, интегралы).
150 826272
>>26250
С Энгельсом ясно, а теорминимум чем тебе не угодил?
151 826275
>>26267
Это неофициальная вики, в которую Энгельса вставили по фану вместе с несколькими другими рофлокнигами. Интересно у тебя информация оттуда превращается в "советуют читать на парах".
152 826276
>>26272
Три года люди типа "учат матан". Ни единого упоминания дифференциальных форм и теории де Рама. В чём смысл?

Лучше бы вводный курс теорката прочитали. А то во всей их википедии даже функторы не встречаются, зато "проекторы Шаудера" есть.
153 826277
>>26275
Я еще добавлю, наверное, что такой подход абсолютно ничем не отличается от подхода людей, которые видят вышкинский майнор по гендерстадис и начинают орать про то, что во вшэ заставляют всех быть гомогеями.
154 826280
>>26276

>Три года люди типа "учат матан". Ни единого упоминания дифференциальных форм и теории де Рама. В чём смысл?


Как будто упоминание дифференциальных форм и теории де Рама привнесет смысл в трехгодичное изучения матана.
5d201bba1972fa013255ffd442e1004e.jpg53 Кб, 600x600
# OP 155 826283
>>26280
Если уж преподавать матан, то хотя бы не на двести лет устаревший. В том виде, как оно там сейчас, - это совсем анекдот.
156 826297
>>26267
>>26250

>Если в ИТМО на парах матана рекомендуют читать Энгельса - это не байт, а ценная информация об уровне преподавания математики в ИТМО, ящитаю.


А вот это:

> Гиляровский В. - Москва и москвичи


Тебя не смущает в списке?
# OP 157 826299
>>26297
Энгельс смешнее.
158 826316
>>26283
Ну охуеть, давайте сейчас всем прогерам загрузим анализ на многообразиях, лишь бы полутора сектантам угодить. Я думал, что всем этим обычно на первом курсе матфака людям удается переболеть, но у некоторых, видимо, оно в хроническую форму переходит.
159 826318
вот кстати главный ответственный за анализ в итмо

mathnet.ru/rus/person23481
# OP 160 826320
>>26316
Я не вижу причин учить прогеров многообразиям. Но раз уж их почему-то принудительно учат "матану", то можно хотя бы делать это нормально, а не через одно место.
161 826335
>>26318

>главный ответственный за анализ в итмо


Кохась ведет что-то на матмехе спбгу в петергофе. В итмо его пригласили читать матан ктшникам.

>>26320
Прогреров обычно учат "калькулюсу", этакому подобию матана, где собраны основные факты и основной упор делается на практическое примение (ну там интеграл взять или производную), а не пониманию теории и доказательству теорем.
На ПМИ (прикладная математика и информатика) иногда пыатются всовывывать полноценный матан, но студенты хуй кладут на изучение этого предмета.
162 826490
>>26335
Непонятно, зачем это делают.
163 826527
>>26490

>зачем это делают


На всех технических специальностях учат матану или калькулюсу. Программисты по традиции относятся к "технарям", поэтому и читают им такой матан.
Всем очевидно, что 99% программистов никак не будут использовать математику вне вуза, поэтому и преподают таким вот образом.
164 826540
>>26527
Бессмысленная трата времени и денег.
165 826543
>>26540
Ты сейчас описал жизнь
1575982872-13df062c4e3162f6113956f3cfb979b2.jpeg181 Кб, 936x821
# OP 166 826544
>>26543
У программистов есть рационализация: якобы, матан "развивает мозг".
16094176236430.jpg37 Кб, 482x1079
167 826605
Сап, у меня послезавтра экз по матеше, 1 сем, посоветуйте какой нибудь учебникчтоб можно было производные похуярить. На маспрофи всё прорешал, но хотелось бы чего то большего(больше примеров). Важно чтоб в конце книжки были ответы, пытался по учебнику Рябушко но там ответов нету и хуй знает правильно решил или нет.
168 826619
>>26605
Лоран-Шварц
169 826622
>>26605
Экзамен по взятию производных что ли? Ну задачник Кудрявцева 1 том возьми.
170 826624
>>26619

>Лоран-Шварц


Шварц хуйня, бери Лорана - не прогадаешь.
Зорич 171 826629
>>26605
Зорич, Математический Анализ, том 1.
Зорич, Математический Анализ, том 2.
В этих книгах ты найдешь ответы на все вопросы и даже больше.
172 826638
>>26605
Зорич
173 826645
>>26629
Пиздёж. Там пучков нет. ПУЧЧК ГРООТ
174 826647
>>26645
Пучки не нужны для специализации на кафедре математических и компьютерных методов анализа на мехмате МГУ. А вот анализ знать надо по классическим учебникам: Садовничий-Чубариков-Архипов вам как пример.
175 826763
Посоветуйте одну книгу быдло-школьнику-одиннадцатикласснику чтобы понять что такое математика
BC41490721697.jpg30 Кб, 250x354
# OP 176 826780
177 826805
>>26780
Эту книгу даже Вербицкий не понял
DB076F08-F4F4-46C6-8153-96CBCEAC14E6.jpeg49 Кб, 362x512
178 826807
>>26763
В этой книге ты найдешь ответы на все вопросы и даже больше.
доказательства из Книги.jpg18 Кб, 370x500
# OP 179 826845
>>26805
Запрос был про одну книгу. Так что нужно было выбирать.
Лично мне больше нравятся книги в духе пикрелейтед, но они меньше подходят под реквест.
180 826882
>>26845
так школьник то не поймёт что там написано
181 827224
>>23017 (OP)
Аноны которые вкатываются сами по себе без универов и репетитров. Как вам заходит материал? Как вы усваивете его?
Я вкатываюсь в физику по книгам. В лучшем случае понимаю процентов 40. Из этого много забывается. Сейчас например дочитываю книгу по квантовой механике. Есть интересные идеи и я для себя подчерпнул новое, но понимаю что книга еще не прочитана (2 главы остались) а какие то вещи уже успели забыться.
Думаю с математикой еще хуже ситуация, физика всё таки более прикладная наука.
182 827281
>>27224
перечитывай, конспектируй, пересказывай параграфы
183 827295
Какие книги по математике годные есть для инженеров? интересуют диффуры, мат/векторный анализ, линейная алгебра.
184 827296
>>27295
Не в тот тред. Тут чистая.
185 827302
186 827308
>>27296
ну блин
187 827342
Всем привет, пацаны, такая хуйня: сейчас в 11 классе и уже проблем с поступлением впринципе нет, что позволяет мне выделить несколько часов в неделю на изучение не школьной математики и вот не знаю, с чего начать? За пределами ЕГЭ и некоторых несложных перечневых олимпиад по типу пвг или шизтеха знаю что-либо вершками, а поступать собираюсь на нормальный математический факультет и в последствие возможно даже связать жизнь с математикой. Мне начинать смотреть вузовские лекции или начинать с книг из подготовки матфака вшэ, по типу книг по теории множеств и прочему?
188 827401
>>27342
Походи в НМУ, будешь в числе 2-3 школьников
189 827407
>>27401
А я думал, что хуйня про то, что москвичи ведут себя так, как будто все живут в Москые - пиздаболия и миф. Ну мне до НМУ примерно 4 тысячи километров
190 827421
>>27407
Они записи лекций делают, также в прошлом году все занятия онлайн были
191 827787
>>27295
Анго Андре. Математика для электро- и радиоинженеров
192 828316
>>26544

>матан "развивает мозг".


Так и есть. Но не только матан.
193 828326
>>28316
Что это значит? Как опровергнуть эту гипотезу? Поппер.жпг
194 828327
>>28326
Все давно уже строго научно доказано
https://en.wikipedia.org/wiki/Neuroplasticity
1dca12205f40dc85e880bbd4c4ab92de.jpg62 Кб, 608x608
195 828373
>>28316
Факультеты программистов нужны, чтобы человек после них становился программистом топового уровня, способным на научную работу. Не надо, пожалуйста, уподоблять университеты кружкам саморазвиванцев.
196 828662
>>28373
В том-то и дело, что вузы себя рекламируют таким образом, что студенты туда идут, на какую-нибудь ПМИ, думая, что это дохуя им нужно для работы, именно обычной работы программистом в компании.
В итоге потом до них доходит то, что учебный план типичных ПМИ, особенно в крупных вузах - это хуйня для подготовки научных кадров, для академической работы в CS, в околоматематических дисциплинах CS и т. д., что они чуть ли не 4 года учат какую-то нахуй не всравшуюся им хуйню, и максимум, что полезного они могут из этого всего взять - это пару на самом деле нужных учебных курсов, стажировку от вуза и саму корку.
И в конце концов до людей доходит, что, чтобы вкатиться в какой-нибудь веб, можно просто подучить пару фреймворком и идти работать, а не дрочить какую-то хуйню 4 года. Или, чтобы стать какой-нибудь распиаренной дата-макакой, можно просто подучить какой-нибудь р, питон, фреймворки нужные, взять пару курсов на курсере по терверу с матстатом, и пойти на галеру, не дрочась 4 года, блядь, в вузе.
Говорю, потому что буквально дохуя таких знакомых есть, которым, чтобы это понять, понадобилось 4 года на ПМИ/ПИ, потому что вузы не гнушаться зазывать и наебывать, мол, "готовим инженеров-разработчиков".
197 828894
Есть ли сайты, где можно померить свои силы в математике?

Пока знаю только khanacademy.
198 828925
>>28662
Анон, а что это за дисциплины для cs на пми-то?
199 829122
>>28662
Как ты станешь machine learning engineer без института?
Вот распиши по пунктам, мне очень интересно.
200 829127
>>29122

>Как ты станешь machine learning engineer без института?


А что ВУЗ даёт для становления "machine learning engineer"? Матан и линал (в том объеме который проходится на первом курсе, больше для мла не надо), тервер со статистикой, и навыки программирования. Что конкретно из этого по твоему мнению нельзя осилить самостоятельно?
201 829158
>>29127
Как минимум это общение со спецами в отрасли.
МФТИ сейчас мутят бесплатные курсы по этой теме, но без личного общения с носителями знания всегда будешь в попе.
202 829189
203 829195
>>29158

> общение со спецами в отрасли.


Нахуя это нужно для вката? Да и никто не мешает ездить на хакатоны и конференции и общаться там.
204 829220
>>29195
Одно дело, когда тебя прямо учит мастер в отрасли, а другое с кем-то кофеек попить на конференции.
205 829222
>>29189
И ты уверен, что у него нет Вышки?
206 829233
>>29222
Там же написано что есть по финансам. Тем не менее смог осилить "сложнейшую математику" для ML и за год до 18 места на кэгле добраться. А тут многие свято уверены, что без красного диплома мфти и phd в машин ленинге ловить нечего.
207 829242
>>29233
Да тут где-то два штатных шизика эту хуйню несут, какие "многие"?
Любому человеку понятно, что МЛ - это не рокет сайнс, и туда вкатиться спокойно самому можно, если захочется.
У этих шизов, наверное, чтобы в веб вкатиться, тоже обязательно нужно 4 года учиться и "общаться с профессионалами", иначе просто никак.
Шизоиды, сэр.
208 829244
>>29242

>чтобы в веб вкатиться, тоже обязательно нужно 4 года учиться и "общаться с профессионалами",


Да, шизик. Компьютерные системы, дизайн требуют минимум 4-ёх лет учёбы. Но для сайта-визитки хватит и месяца, да. Обычно для саморазвиванцев этого приемлемо, на пиво и книги властелин колец хватает
209 829247
>>29242

>у этих шизов, наверное, чтобы в веб вкатиться, тоже обязательно нужно 4 года учиться и "общаться с профессионалами"


Ты думал, что ты пошутил.. --> >>29244
Хотя, скорее всего, конечно, толстота.
210 829249
>>29244
Если кто-то хочет быть на дне с пивом и властелином колец, то пусть он сразу признается, а не строит из себя программиста.
211 829252
>>29249
Почти все программисты пьют пиво, играют в фортнайт и киберпанк, смотрят марвел, властелин колец и сидят в тиндере.
212 829255
Считаю, чтобы вкатиться в какой-нибудь фронтэнд или бэкэнд, нужна минимум степень PhD из топ-100 вуза в мире, желательно из MIT, конечно. Без этого вкат невозможен, без этого вы просто хуйня, которая семки грызет, пиво пьет с воблой, книги толкина читает и в танки играет, уяснили?
213 829259
Советую вам почитать дневники вкатывальщиков из /dr. Люди прекращают ботать уже через неделю.
214 829261
>>29255
ЕСТЬ ПРОБИТИЕ
avatars-zSTp5Pd72Va4NrIy-ssE7oQ-t500x500.jpg44 Кб, 500x500
215 829287
>>29255
В чем проблема с книгами Толкина? Ma naccë alasailar?
216 829301
>>29287
Читаешь Толкиена под пивасик и будешь вебмакакой, пока нормальные пацаны вкатятся в топовый ВУЗ на Machine Learning Engineering.
217 829302
>>29259
Это потому что они в соло вкатываются, а надо вместе с кем-то, чтобы обсуждать вопросы и проблемы.
Я бы запилил какой нибудь чатик для общения, но сам пользуюсь только дискордом.

Лично у себя заметил сбитый режим и плохие навыки планирования. Если уделять по часов 4 в день, то через полгодика можно вкатываются в профессию. Но то лень, то депрессия, то ещё какая хуета.
218 829347
>>29301

>вкатятся в топовый ВУЗ на Machine Learning Engineering


И будут по вечерам пить пиво под Толкина.
219 829355
>>29347
Будут пить вино и читать Античных греков.
/пофиксил
16106025584410.png8 Кб, 227x222
220 829420
>>23017 (OP)
Хочу вкатиться в матешу, ну как, я уже студент в айти, но плохо запомнил 9-10-11, есть какие годные материалы по этим классам? Олимпиадные тоже готов позадротить, подскажите анончики
shoto-todoroki-sleeping-mask-large.jpg80 Кб, 600x600
# OP 221 829547
>>29301
Umë issë parë aumar.

>>29420
Скорее всего, тебе будет достаточно пособий для подготовки к ЕГЭ.
Только вот что-то мне не кажется, что для того, чтобы читать учебники по теории множеств или линейной алгебре, нужно знать что-то из 9-10-11 классов школы.

http://gen.lib.rus.ec/book/index.php?md5=D105C266D0A2BB547C9F24D31EFE6BB9
Вот этот справочник советуют.

Чтобы получить доступ к либгену из России, можно использовать
http://freeopenvpn.org/
222 830282
поясните за математические модели. как в них вкатиться?
223 831258
Посоветуйте книг для знакомства с вузовской алгеброй(пока разбираюсь с Кострикиным), матанализом, теорией вероятности(что кстати нужно знать для хорошего освоение теорвера?).
И что можно по дискретной математике почитать/порешать? Решал только комбинаторику Виленкина и введение в теорию графов Уилсона.
E7E7A090-C01E-42F6-8E28-BFC1C035D2C4.jpeg75 Кб, 530x711
224 831263
>>31258
Читай Зорича, Математический анализ, там ты найдешь ответы на все вопросы и даже больше.
20210207180416.JPG67 Кб, 531x543
225 831281
>>31263

>Читай Зорича, Математический анализ, там ты найдешь ответы на все вопросы и даже больше.

226 831282
>>31281
Кто это?
227 831283
>>31282
Слева Берчун, справа Вадик (фанат Зорича)
228 831372
>>31263
Там пучков и категорий нет(
229 832419
>>23017 (OP)
Зачем математики дрочат на формальные доказательства? Ведь в любой системе аксиом есть неопределяемые(эмпирические) понятия, значит и любое доказательство является эмпирическим, так к чему его усложнять формализмом?
230 832541
>>32419
удали сообщение не позорься....
231 832869
>>32419

На доказательства. На формальные дрочат формалисты и матлогики.

Формализм позволяет вывести что-нибудь неочевидное из очевидного. Если действовать по интиуции, выведешь хуйню.
232 833351
А насколько вообще большая, по размеру, математика? Вот в ВУЗах тебе дают всю математику, что сейчас известно науке или всё равно остаётся какая то не изучаемая в вузе часть? Сколько по времени нужно, что бы изучить всю математику?
233 833390
>>33351

>в ВУЗах тебе дают всю математику, что сейчас известно науке или всё равно остаётся какая то не изучаемая в вузе часть



В вузах дают знания из математики на момент конца 19 века. И то, иногда не полностью. Остальное либо изучается на спецкурсах и спецсеминарах, либо не изучается вовсе.
234 833412
>>33390
Дофига всего получается за рамками программы остаётся.
235 833518
>>33412
Да, это очень крупная наука, как и любая другая.

Всю математику не изучишь, там ДОХУЯ. Физику тоже всю не изучишь, там тоже ДОХУЯ.

Очень базу в вузе дадут. Если будешь постоянно ходить на научные семинары и все понимать, то за бак с магой с можешь примерно понять, что происходит у тех чуваков, что семинары вели. На данный момент. Но без деталей. Вот в лабу пойдешь работать, тогда уже можно говорить, что в какой-то области начал шарить.
236 833541
>>33518
Чтож спасибо, хотя бы узнал примерные размеры математики
Matan.png84 Кб, 1366x736
237 833909
Двач! Спасай нужна Помощь!

Задание никак не могу решить для универа. Сказывается пинание хуев на уроках матемы с 8-го класса.

Нужны решения с шагами. Заранее благодарю отважных волонтеров. С меня как обычно.
238 833910
>>33909
Да, сейчас проебу время, чтобы бесплатно расписать решения с шагами для бездаря, который зачем-то поступил в университет с 8 классами образования и там не смог за семестр вызубрить алгоритмы решения базовой матеши
239 834030
>>33909
Бери серию учебников мордковича с 7 класса и читай всю школьную программу. Внатуре чётко пишет! В учебниках нет упражнений, ты их и не решай, а анализируй только примеры данные. Быстренько подтянешься и всё будет нормально
240 834031
>>34030
Примеров в Учебнике нет, но есть дополнительно задачник, толщиной сопоставимый с ним, лол.
241 834070
>>33909
Решу все кроме 1 номера за 500р
242 834074
>>34070
А какая сложность с 1 номером?
243 834077
>>34074
Никакая, просто хочу заработать и хоть немного поднасрать
kakayazhetimraz.webm258 Кб, webm,
1280x720, 0:03
244 834089
245 834740
Только не обкладывайте хуями. Я НЕ ПОНИМАЮ эээ смысла этих функций, производных, дифференциалов, лимитов. В школе и биологическом вузе решал задачи и не было проблем. А теперь решил копнуть поглубже и всё. Стенка. Не понимаю. Что читать - смотреть, чтобы и дауну было понятно.
246 834742
>>34740
Трушин
247 834744
>>34740
Классический учебник по матану? Цели какие у тебя?
248 834745
>>34744
Понимать и применять эти штуки.
Извините за перевернутость.
249 834747
>>34745
Пределов не увидел. По диффурам на скринах самые основы это любой учебник универовский для вузов попроще, можешь задачник Филиппова порешать если хочешь прям руку набить. Это классика, по нему даже на мехмате МГУ контрольные дают и там несложно. По пределам, функциям в Фихтенгольце очень хорошо и понятно написано.
250 834749
>>34747
Не слушай этого пизданутого, заходи на матпрофи. Того уровня, который там даётся тебе хватит с лихвой.
251 834750
>>34749
Матпрофи это все-таки для совсем дебилов типа тебя, которым надо типовичок решить и забыть. А в учебниках строгое математическое изложение.
252 834752
>>34750
Зоричешизик, спок.
253 834754
>>34750

>в учебниках строгое математическое изложение


Строгое изложение требуется только аутистам. Для реальных дел нужно иметь интуитивное понимание. В хороших вузах типа MIT, МВТУ как раз такой интуитивный подход.
Но я не защищаю матпрофи. Матпрофи вообще не даёт никакого понимания, это просто алгоритмы для решения контрошек.
254 834755
Поясните за образование по программе "Математика" vs "Прикладная математика".
Есть тезисы, которые я слышал довольно часто:
1) чистая Математика это бесполезная дрочка гомологий, которые нахуй никому не нужны в реальной жизни
2) чистая Математика раскачивает нейронную сеть в мозге за счет чего потом легко и быстро можно освоить любую другую науку
3) университетские программы по чистой Математике включают все что есть в программе Прикладной математике плюс много чего еще. Поэтому поступая на чистую Математику не ошибешься в любом случае
4) образование по чистой Математике наиболее универсально, потому что дает тебе возможность работать в том числе в тех сферах IT где нужны глубокие знания в математике. Хочешь работать в дата сайенс или machine learning? Изи. Хочешь в ШАД? Тебя там ждут, а обычные информатики не смогут даже двух заданий решить по математике. И так далее.

Поясните пожалуйста по поводу этих пунктов. Разрываюсь между Математикой, Прикладной математикой и ПМИ (ФКН) во ВШЭ.
255 834757
>>34754
Ну он все-таки универ закончил. Хотя дело его конечно, матпрофи так матпрофи.
Погоди-ка...

>В хороших вузах типа MIT, МВТУ



ВАДИК, ПОШЕЛ НАХУЙ!
63CE0881-AED7-40EF-AEEB-4BAF2BECCA1A.jpeg197 Кб, 1280x1024
256 834761
>>34755
Просто выбирай инженерную или физическую программу.
И не во ВШЭ, а в каком-нибудь хорошем вузе.
МГУ, МФТИ, МВТУ к твоим услугам. Точнее это ты к их услугам.
257 834763
>>34750
Я понимаю, что ты учишься на втором курсе и в тебе бурлит юношеский максимализм. Но твой способ изучения вообще не знакомых для человека вещей - полная хуита.

Открывать с нулевыми знаниями такой большой учебник и читать его - это пиздец длительный, а значит изматывающий процесс. Скорее всего чел нихуя не поймет и забросит эту тему, толком не разобравшись после пары недель ковыряния.

По моему опыту самым лучшим способом быстро разобраться в чем-то - сначала пройтись по верхам, поняв самые общие идеи. И только потом, когда уже сформируется представление о том, что конкретно тебе неясно - копать вглубь. Так что матпрофи это идеальный сайт, чтобы получить представление о дифурах и прочей хуйне.
258 834764
>>34763

>Я понимаю, что ты учишься на втором курсе



Я закончил давно на ебало твоей мамаше.

>Открывать с нулевыми знаниями такой большой учебник и читать его



Весь зачем читать? Главу про функции и пределы осилит и дальше пойдет.
259 834766
>>34764

>Я закончил давно



Печально, что не научился учиться
260 834767
>>34755
На ПМИ ФКН при желании сможешь с ног до головы обмазаться майнорами по чистой математике, а потом специализироваться на теоретической информатике. Адепты Димы Павлова будут считать тебя второкультурщиком, но не похуй ли? Всяко лучше, чем на матфаке занимать место человека, который, может, с детства этим всем горит. А про ПМ надо понимать, что это, мягко говоря, не совсем вышка.
261 834769
>>34767

>Всяко лучше, чем на матфаке занимать место человека,


Всем похуй на этого человека.
А школьник наверху понятия не имеет о чём говорит. Ему лучше действительно пойти в МИЭМ качать нейроны, потому что в другом месте он отлетит нахуй мгновенно.
262 834772
Лучше уж нахуй, чем в МИЭМ, имхо конечно.
мимо
263 834773
Да и отлететь мгновенно нахуй тож полезное дело ведь, реальность лучше одуплять будешь, вот и прокачка нейрончиков ахаха
264 834775
>>34773

>проебать минимум год, заработать расстройство сна и другие психические отклонения, в итоге стать шизом-ВПЛом


>тож полезное дело

265 834777
>>34775

>минимум год


Да какой год? Если этот ебанат пойдет на ФИВТ, как он хочет, то отлетит он еще до первой сетки, но скорее всего сделает это переводом через пару недель обучения. Раньше таких ходаков из топВУЗов дохуя и больше было, так что если лавку не прикрыли, то без особого вреда для себя человек осознает, что 'прокачку нейросети в голове через систему физтеха' он в гробу видал и впредь о ней даже не вспомнит.
266 834781
>>34766

>что не научился учиться



Этот мощный вывод основан на том, что я порекомендовал учебники, а не матпрофи?
267 834784
>>34781
На том, что ты вообще не понимаешь, что начинать изучать математику с нуля с фихтенгольца это плохая идея. Видимо тебе не приходилось по работе усваивать много всяких дифурчиков в сжатые сроки.
16093355793300.jpg26 Кб, 437x563
268 834786
Матпрофи - весьма полезный ресурс. Он хорошо составлен, его удобно использовать для подготовки к коллоквиумам. У нас на Матфаке многие пользуются.
269 834787
>>34784
Але, он универ закончил. Фихтенгольц это не Зорич, там доступно изложено.

>Видимо тебе не приходилось по работе усваивать много всяких дифурчиков в сжатые сроки.



Так ты тупой просто.
270 834791
>>34787
Он заканчивал биологический, думаю математика там читалась а уж тем более усваиваласьдаже не на уровне фихтенгольца.

Представь, что ты приходишь в секцию бега. И вместо того, чтобы переодеться, послушать про технику бега, как стопу ставить, ты сразу бежишь полумарафон. Это бессмыслено, и даже вредно. В учёбе, как и в спорте, должна быть система, или надорвёшься.

>Так ты тупой просто.


Как скажешь)
271 834795
>>34791
Ты тян что ли? Что за ебанутая логика. Классический учебник по матану это и есть "послушать про технику бега" в твоей кривой аналогии. Функции и пределы это первые 2 месяца на первом курсе.

>)



Нахуй пошла.
272 834798
>>34795
Зоричешизик, спок.
273 834807
>>34772
>>34767

МИЭМ же входит в состав ВШЭ и ты получаешь в итоге диплом Вышки?

>>34761

>>Просто выбирай инженерную или физическую программу.


>>И не во ВШЭ, а в каком-нибудь хорошем вузе.



Во ВШЭ кстати есть физфак, ему 3 года всего и еще не было выпуска, но судя по всему очень крутой. Программа Математики в отличии от инженерки мне нравится универсальностью. Ты можешь потом перекатиться куда хочешь (по идее), хоть в аналитику в банке, хоть куда. В том же шаде пацанов гасят именно задачами по математике.
Я просто понял что в разных областях я буду упираться в знания по математике, поэтому что матфак хорошая база для будущего.

>>И не во ВШЭ, а в каком-нибудь хорошем вузе.



Да ну брось. В Вышке же топовая математика.
По крайней мере у меня друг (на год старше) поступил после школы на мехмат МГУ и матфак Вышки. В итоге отказался от МГУ. Родители хотели его в дурку отправить, типа "Как так? Ты отказался от МГУ?". В их совковом представлении лучше МГУ ничего нет. Но после элементарных разъяснений поняли что все нормально и ВШЭ збс.
274 834872
>>34807
Кто-то может расшифровать этот шизоидный бред?
275 834874
>>34872
Да это либо шприцетык, либо его эпигон, пришедший потроллить. Никто уже не будет в 2021 году всерьёз двигать телеги про универсальность матфака и его пользу для работы аналитиком.
276 834875
>>34874

Правильно ли я понимаю, что Прикладная математика и информатика самое универсальное?
А просто Прикладная математика для айтишника лучше чем ПМИ или что?
277 834876
>>34875

Да. Пми определенно лучше чем чистая математика с дрочкой всяких гомологий, которые как ты правильно сказал в реальной жизни нахуй никому не всрались. На пми дают как раз только тот блок прикладной матеши которая реально используется без всякой дрочки 8-мерных пространств.
278 834877
>>34875
Лесное дело самое универсальное. Сотни вариантов дальнейшей работы по специальности, от применений программирования в лесном деле до суровой мужицкой вахты или работы в министерстве, плюс всегда можно в Канаду переехать, или в Европу по линии экологии. Плюс остаются перспективы в задротском консалтинге и аналитике, где ничего кроме базовых навыков по статистике не надо. Так что, анон, хочешь стать настоящим универсалом - поступай в СПбГЛТУ.
image.png2,2 Мб, 1280x960
279 835070
Добрые люди сделайте мне работу по топологии прошу!! Я заплачу 200 рублей и в будущем буду еще подкидывать задания!!!
280 835097
>>34877
сам по нему отучился? как там с трудоустройством? кем работаешь?
281 835104
>>35097
Нюфаг, не ведись на местного шиза.
8FE95F9C-0149-4B77-8E88-B1139B13F986.png314 Кб, 1000x667
282 835110
>>35104
Почему шиза? Он прав так-то. Лесное дело даёт универсальный инженерный базис, там есть продвинутая математика, механика, сопромат, химия, электрониуа.
Лесничество сейчас связано с геоинформационными системами, то есть хороший лесничий должен разбираться в обработке данных (дата сайнс), компьютерах, работе со спутниками, радиоустройствами. Это тоже изучают в рамках специальности.
В Сибири деревообработка и дереводобыча процветает, там на вахте легко заработать миллион+ рублей за несколько месяцев работы.
Про консалтинг тоже прав. В проектах, которые связаны с лесной отраслью, нужны люди, которые шарят в этом. И таких проектов в богатой древесными ресурсами России полно.
Даже в науке есть куда развернуться, гугли Wood Science. В данный момент это крайне недооценённая ветвь, даже учитывая огромным спонсированием от эко-фондов и футуристичных организаций с долгосрочными проектами в сфере инженерии.
Вот небольшой список перспективных предметов для изучения в древесной науке:
-дендрохронология
-искусственная древесина
-ускорение роста древесных
-целлюлоза
-полифункциональные материалы из бамбука
-деревянные композиты
-различные многослойные панели
-эко-обработа древесных
-молекулярная инженерия дерева
Короче, ценность и востребованность лесотехнических специальностей будет только возрастать со временем. Сейчас ситуация в области напоминает ситуацию с компьютерами в 50-х годах прошлого века. Тот, кто поймёт это и рано успеет вкатиться сюда, рискует получить большое состояние.
283 835111
вообще дерево - это материал, который никуда в 21 веке не уйдёт. более того он станет доминирующим, древесные биокомпозиты показывают просто впечатляющие результаты, это буквально наше будущее.
сам заканчиваю не лестех, а РК5 бомонки, в лестех пойду в магу скорее всего. тема меня очень интересует, много слежу за ней.
284 835113
Бля, ладно ебанат срет про лес, но можно хотя бы пасты новые генерировать, а не копировать старое?
285 835115
>>35113
Шизоид, я просто частично скопипастил свой старый ответ (список областей в Wood Science и планы после выпуска), т.к. его потёрли в другом треде.
286 835319
Почему элементраными функциями считаются степенная, показательная, логарифмическая? Чем это обусловлено???!!?
287 835449
>>35319
Потому что это элементарные функции.
288 835464
>>35319
Так исторически сложилось. Реально - это просто частные случаи аналитических функций, первые открытые.
289 835470
>>35319
Всё из-за особенностей их интегрирования/дифференцирования и в связке с физическим смыслом.
В общем, корни этого в истории науки.
image.png1 Кб, 273x42
290 835549
Как получилось то что справа?
291 835550
>>35549
Что за картофан?
A773EFE8-5437-46E0-A33C-BD924062D5C0.jpeg2,1 Мб, 4032x3024
292 835551
293 835553
>>35551
Спасибо
294 835699
>>34877
>>35110
>>35111
Опа, Вадик новый форс придумал.
295 835703
>>34807

>есть физфак, ему 3 года всего и еще не было выпуска, но судя по всему очень крутой


>очень крутой


Двойное чаепитие. Делать лабы с помощью пустых банок "монстер энерджи" это очень неплохо. Не уверен, правда, насчет консервных банок - прошу шарящих людей пояснить, реально ли использовать для выполнения лаб пустые банки консервов "Главпродукт".

https://habr.com/ru/post/451316/
296 836049
>>35703
Пидорах, каково тебе осознавать, что ты никогда не будешь учиться в единственном вузе РФии, где используются практики Успешных Западных Стран? Каково тебе учиться в прогнившем совковом вузе, построенным сумасшедшим грузином-кровопийцей? Каково тебе осознавать тот ужасающий факт, что ты навеки останешься в Пидорахии, в то время как студенты ВШЭ активно находят спасение за рубежом? Понимаете, пидараны, вам никогда не тягаться со студентами ВШЭ, которые победили в генетической лотерее, имеют либеральные ценности и устремлены на Запад. Как бы ты не ботал, сколько бы энергии не прожигал на зубрёжку очередного томика "Марксистской механики" за авторством Сталена и Ландшвица, вы никогда не сможете опередить студентов ВШЭ чисто биологически. Да и в ВШЭ, прямо скажем, связей с Западом намного больше, чем в какой-нибудь совковой дыре, намного. ВШЭ - идеальный трамплин для юных гениев в Европу и Новый Свет.
297 836085
Посоветуйте учебник по матстату для даунов. Знаю про биномиальное распределение, но не знаю, чем отличается PDF от CDF.
298 836161
>>36085
Поищи что-нибудь вроде "математическая статистика для социологов", точно найдёшь.
299 836204
>>36085
Читай Зорича, там все написано
300 836207
>>36204
Согласен. Особенно во втором томе...
301 837178
Я так понимаю тут говорят только про высшую математику для институтов ?
302 837182
>>37178
А что в школьной обсуждать можно?
303 837186
>>37182
Ну так-то да
304 837367
>>37182
Почему иррациональные числа нельзя представить в виде целой дроби
305 837397
>>37182
5^3/2 - это квадратный корень из 5^3.
А как определяется 5^пи?
Безымянный.png3,1 Мб, 1916x1032
306 837440
Хелп, что за книга?
307 837714
Можно ли определить уравнение функции по её графику? Чтобы нарисовать любой рисунок, даже слово написать какое-нибудь и получить для этого рисунка формулу? Или не для каждого графика они есть? если есть, это наверное задача, в которой проверить ответ на порядок проще, чем получить его?
308 837717
>>37714
Аппроксимация, интерполяция. Учебник по численным методам ищи.
309 837726
МАТЕМАТИКИ СПАСИТЕ С РАБОТЫ МОГУТ УВОЛИТЬ НАХУЙ

Если 0,6=1650, то 0,5= СКОЛЬКО?
310 837733
>>37726
x = 1650 * 0.5 / 0.6
311 838399
Хочу в свободное время решать матан, но начать с самых азов. Возможно, уже были подобные вопросы, но все же. Какие учебники наиболее доступны для понимания?
312 838423
>>38399
Зорич
313 838658
>>37726
1375 вроде
314 838660
24 совершенное число? Он ведь равен сумме своих делителей (1+2+3+4+6+8)
315 838663
Сорян, учитывается сумма всех делителей
316 838664
sage 317 838931
>>23017 (OP)
Аноны, много ли физики на математике или прикладной математике? Что там именно из физики, какие темы, насколько глубоко, есть ли лабы как в школе, т.е с измерениями, рассчётами и если где-то проебался то лабу не сдал?
Прошу поясить за это, диваны мимо плз.
318 838940
>>38931
На программах чистой математики (матфак ВШЭ, чебышевка) физики нет вообще никакой и ни в каком виде.
319 838942
>>38940
Ага, конечно. Даже механики нет? Не вешай людям лапшу на уши.
320 838964
>>38931
программу открой и посмотри
321 838968
>>38942
Это просто дань уважения Арнольду и его взгляду на дифуры, а также способ хоть как-то трудоустроить Пятова. К физике как у физиков не имеет отношения. Не веришь - посмотри первый том ландавшица Сивухина и сравни.

>>38964
У матфака нет "программы". Студент сам выбирает себе курсы. За исключением нескольких обязательных.
322 838970
>>38968
ну тогда выбери себе или наконец напиши в вк какому нибудь студенту
brav.png275 Кб, 991x821
323 838973
>>38970
Я не выбираю, я отвечаю на вопрос анона (для ясности - учусь на мф).
sage 324 839016
>>38964
Ты что даун сцука, я же ясно написал

>диваны мимо плз


Хуево тебе в жизне, маня, если даже простой текст не осиливаешь. Может ты чурка? Тогда прости, но лучше учи русский язык. Без него никак.
>>38940
В спб есть 2 семестр мат физики и семестр физики. В вшэ есть механика, судя по википедии это раздел физики.
325 839083
>>39016
да я чуркабес. вопросы?
326 839086
>>39016

>судит по википедии


>диваны мимо


И ещё смеет обзывать чуркой кого-то. Феерично просто.
327 840066
>>23017 (OP)
привет опчик, тут такая штука, что я иностранец, а ты, как я понял, с матфака ВШЭ. У меня тут появился вариант залететь на математику к вам со скидкой 70%, но проблема в том, что я понял, что люблю математику и то, что поступать на матфак рационально слишком поздно (в нячале года) и с того времени мой уровень математики вырос только до уровня решения всего ЕГЭ и Физтеха, в связи с этим я не уверен: выдержу ли я впринципе уровень нагрузки и сложности матфака, не слечу ли я со скидки и не отчислят ли меня? Знаешь ли ты вообще о подобных моему кейсах?
328 840075
>>40066
Учеба на матфаке довольно непростая, и математика, которую тут изучают, на ЕГЭшную не похожа. Можешь оценить свои силы здесь: https://math.hse.ru/bac1-1920

Вот буквально самая первая лекция по анализу:
http://vyshka.math.ru/pspdf/1920/calc1/Lect1.pdf
Если у тебя есть какие-то трудности с тем, чтобы прочитать и понять эту pdf за два часа, то, скорее всего, тебе будет сложно вписаться в темп матфака.

Вот самая первая домашка по алгебре. Чтобы получить удовлетворительную оценку за семестр, нужно делать не меньше четырёх задач. В неделю таких домашек две (по алгебре; есть ещё и листки по ней, и другие предметы, и разные контрольные с пятиминутками).

Я знаю довольно много историй, когда люди не то что слетали со скидки, но и вообще вылетали с матфака. На втором этаже матфака, рядом со шкафами для буккроссинга, находится учебный офис (что-то вроде деканата в обычном вузе; только на матфаке декан сидит на третьем этаже и редко когда появляется публично, а делами студентов занимаются специально обученные люди). Рядом с учебным офисом в конце каждого модуля (модуль - это что-то вроде четверти в школах) на специальном стенде вывешивают списки с фамилиями студентов, которые отправляются на комиссию для пересдачи заваленного экзамена. Как правило, только половина из попавших в эти списки продолжает учёбу, остальные не справляются. Мне это немножко напоминает проскрипции Суллы, и каждый раз грустно, когда в этих списках появляются знакомые имена.

Оп-галка у меня отвалилась, а ставить трипкод я не хочу. Ответы от моего имени не обязательно написаны мной. Вообще, сейчас нет времени на этот тред.
329 840109
>>40075
окей пасибо оп, лекция и домашки показались несложными, так что буду думать, конечно, попытаюсь еще нагнать уровня матграмотности до поступления и буду решать. альтернативой поступления на матфак ВШЭ, к слову, является мехмат НГУ, потому что физику для МГУ я не знаю, а в других вузах с поступлением иностранца не на бюджет все плохо
330 840111
>>40109
блен перепутал жирный шрифт и скрытый
331 840292
>>37440
Precalculus: A Prelude to Calculus, 3rd Edition 3rd Edition,
by Sheldon Axler (Author)
procrastination4.gif.pagespeed.ce.aALPcttYVS.gif14 Кб, 432x519
332 840343
Как настроиться на это ваше обучение? Я не понимаю как свыкнуться делать это дома, мне атмосфера очень мешает - дом место где я расслабляюсь. Как привыкнуть к этому? Чё вы делаете?
333 840449
>>40343
Чтобы стать математиком нужно для начала сойти с ума
334 840470
>>23017 (OP)
Аноны, подскажите годную книгу по алгебре(возможно с интегрированным курсом ангема), прям такой "золотой стандарт" как Матан Зорича, чтоб я мог иногда возвращаться к ней и вспоминать материал(покупаю физические версии).
335 840492
>>40470
Винберг
336 840505
>>40492
Разве там ангем есть?
337 840507
>>40505
Нету. Но это и не было обязательным требованием.
зорич.png120 Кб, 2049x577
338 840565
>>40470
Матан Зорича не является золотым стандартом ни в каком смысле. Автор пытается закосить под Бурбаки и изложить всё с максимальной общностью и строгостью, но у него не получается. То увлекается естественно-научными аналогиями, то пытается рассказать элементы гладких многообразий. Из-за этого непонятно, на кого рассчитан учебник. Для начинающих он слишком сложен и алогичен. Для людей, уже откуда-то знающих матан, - слишком слаб и поверхностен.

Первый том открывается перечислением аксиом ZFC и теоретико-множественных конструкций. Часть аксиом автор опускает (например, аксиому регулярности), часть рассуждений проводит неверно (в доказательстве существования декартова произведения у Зорича ошибка, по которой даже успел проехаться Вавилов). Ради чего было это перечисление - непонятно. Налить воды, чтобы объём стал побольше?

Язык, который автор предлагает в качестве логической нотации, не есть язык теории множеств. Это просто какие-то стенографические сокращения, а не первопорядковые формулы. Обсуждения логики предикатов у автора нет. Есть лишь таблицы истинности, годящиеся только для исчисления высказываний.

Определение предела в первом томе даётся "по базе множеств". Это определение слишком абстрактно для первого знакомства с понятием предела. Можно было бы понять резоны автора, если бы это определение было дано раз навсегда на весь последующий текст - но нет, автор постоянно переопределяет предел. Нотация при этом переусложнена. Пикрелейтед: надстрочные индексы, подстрочные индексы, диакретика, два типа зависимостей, первопорядковые символы, определяемый символ, метаязыковой символ дефиниции. Как это предлагается осознавать новичку при первом чтении, мне не ясно.

Собственно анализа в этих книгах мало. Нет обсуждения интересных объектов (вроде того, что имеется в "Контрпримерах в анализе"). Нет обсуждения классических для анализа вещей вроде признака Раабе и тем более признака Куммера. То есть это на самом деле не учебник анализа, это учебник чего-то другого.

Много таких замечаний. Я могу целую простыню написать.

>годную книгу по алгебре


>с интегрированным курсом ангема


Ангем - это линейная алгебра. А "алгебра вообще" - это общая алгебра. Как ни крути, нужны две разные книги. Я предлагаю вот эти две:
Шафаревич, Ремизов. Линейная алгебра и геометрия.
Dummit, Foote. Abstract algebra.
зорич.png120 Кб, 2049x577
338 840565
>>40470
Матан Зорича не является золотым стандартом ни в каком смысле. Автор пытается закосить под Бурбаки и изложить всё с максимальной общностью и строгостью, но у него не получается. То увлекается естественно-научными аналогиями, то пытается рассказать элементы гладких многообразий. Из-за этого непонятно, на кого рассчитан учебник. Для начинающих он слишком сложен и алогичен. Для людей, уже откуда-то знающих матан, - слишком слаб и поверхностен.

Первый том открывается перечислением аксиом ZFC и теоретико-множественных конструкций. Часть аксиом автор опускает (например, аксиому регулярности), часть рассуждений проводит неверно (в доказательстве существования декартова произведения у Зорича ошибка, по которой даже успел проехаться Вавилов). Ради чего было это перечисление - непонятно. Налить воды, чтобы объём стал побольше?

Язык, который автор предлагает в качестве логической нотации, не есть язык теории множеств. Это просто какие-то стенографические сокращения, а не первопорядковые формулы. Обсуждения логики предикатов у автора нет. Есть лишь таблицы истинности, годящиеся только для исчисления высказываний.

Определение предела в первом томе даётся "по базе множеств". Это определение слишком абстрактно для первого знакомства с понятием предела. Можно было бы понять резоны автора, если бы это определение было дано раз навсегда на весь последующий текст - но нет, автор постоянно переопределяет предел. Нотация при этом переусложнена. Пикрелейтед: надстрочные индексы, подстрочные индексы, диакретика, два типа зависимостей, первопорядковые символы, определяемый символ, метаязыковой символ дефиниции. Как это предлагается осознавать новичку при первом чтении, мне не ясно.

Собственно анализа в этих книгах мало. Нет обсуждения интересных объектов (вроде того, что имеется в "Контрпримерах в анализе"). Нет обсуждения классических для анализа вещей вроде признака Раабе и тем более признака Куммера. То есть это на самом деле не учебник анализа, это учебник чего-то другого.

Много таких замечаний. Я могу целую простыню написать.

>годную книгу по алгебре


>с интегрированным курсом ангема


Ангем - это линейная алгебра. А "алгебра вообще" - это общая алгебра. Как ни крути, нужны две разные книги. Я предлагаю вот эти две:
Шафаревич, Ремизов. Линейная алгебра и геометрия.
Dummit, Foote. Abstract algebra.
339 840571
>>40565
А какие книги по матану тогда рекомендуешь?
340 840590
Всем привет. Посоветуйте книг по основным разделам математики(матан, теорвер, линал), желательно англоязычные источники, но если есть что-то действительно годное, то можно и на русском. Уходить в чистую математику не планирую
341 840593
>>40571
Я осознаю все минусы Зорича, но всё равно готов порекомендовать эту книженцию. Она займёт заслуженное место на твоём столе или твоей полке. Вообще упомянутые мной недостатки в процессе чтения начинают стираться. Система Зорича развёртывается у тебя в голове, методы математического анализа становятся натуральными и лёгкими и тебе начнёт казаться, что ты уже родился с ними. В такой ситуации все огрехи книги сходят на нет, т.к. твоё восприятие кардинально поменяется.
342 840594
>>40590
Зорич
343 840596
>>40571
Не знаю. Я не особо верю в значимость матана, поэтому не хочу давать советы. Я просто считаю книгу Зорича весьма неудачной. Могу сказать с определённостью, что для решения задач типа Демидовича более полезными окажутся Фихтенгольц (да, я серьёзно) и курс высшей математики Смирнова, а концептуально гораздо более интересными являются "Анализ" Шварца и "Анализ на многообразиях" Спивака. Ну, и ещё Рудин, куда же без него. Вроде бы из основных книжек Рудина по анализу (baby, papa и grandpa) на русский переведена только первая, которая Основы, но точно не знаю. Зорич сакс.
1FBC51DA-880B-45BF-BCFC-9DF44876A289.jpeg433 Кб, 1242x1901
344 840600
>>40565

>Матан Зорича не является золотым стандартом ни в каком смысле. Автор пытается закосить под Бурбаки и изложить всё с максимальной общностью и строгостью, но у него не получается.


какую же хуйню ты несешь, дегрод
никого твое никудышное мнение не интересует
математический анализ Зорича - одна из лучших книг в своей области, подходит как для начинающих, так и для продвинутых, желающих углубиться в математику
f0e59d51b8d6f4598d7b8e448ab96981.png215 Кб, 400x400
345 840604
>>40600

>Архипов, Чубариков, Садовничий


Ещё бы Камынина предложили. Пздц, какой пздц.
346 840607
>>40565
Хуясе. Это копипаста откуда-то или ты сам выдал?
347 840608
>>40593
>>40594
>>40600
Вадик, ехай нахуй.
584ad62ff0e229a25be4d01d.jpg80 Кб, 1200x675
348 840613
>>40571
Я не оп, но сам учил по пикрилейтед книге (фото не мое - нашел в гугле, название "Analysis I,II,III", авторы - Amann-Escher).

Книга очень концептуальная, вроде Шварца, но написана современнее и аккуратнее, на мой взгляд. Идеалом её не назовешь, впрочем (некоторые параграфы все равно неаккуратно написаны). Ещё нюанс - из языков только английский и немецкий, на русский не переведена. Читается не очень сложно. Достаточно много примеров.

Но это только если решил угореть по чистой математике (в противном случае тебе и Зорич не очень нужен).
349 840633
>>40607
Текст мой, но изначально написано в первом треде. Имеешь возразить?
350 840659
>>40633
Нет
351 840742
Поясните за функцию, чтобы перестать видеть её как зависимость y от x в уравнении и увидеть что функция основополагающие понятие! Или если это не функция, то что?
352 840777
>>40742
Функция - тройка (f,X,Y) (пишется f: X -> Y), где X (домен) и Y (кодомен) - абстрактные множества (например, X - множество всех подмножеств множества кватернионов, а Y - множество какого-то количества векторных пространств), а f - "правило", которое ставит каждому элементу x, принадлежащему X, единственный элемент y = f(x), принадлежащий Y. Если существуют y в Y, которые не представляются в виде f(x) (это возможно и очень часто происходит), то могут быть разные функции (f,X) могут определять разные функции (f,X,Y), то есть кодомен - это часть структуры функции.

Для вчерашнего школьника это определение действительно контринтуитивно, и к нему надо привыкнуть путем чтения математической литературы - абстрактной алгебры, линейной алгебры, наивной теории множеств (с неё лучше начать) и строгих книг по матанализу. Иногда может хватить одной теории множеств, но именно наивной - в книгах по аксиоматической ТМ под функцией понимают нечто другое, чем в остальной математике (для специалистов в АТМ кодомены отождествляются с множеством значений почему-то).
353 840779
В целом, понятия о функции в школе и в математике отличаются следующим:
1) Функция f: X -> Y не обязательная числовая - элементы X и/или Y могут быть чем угодно - не обязательно числами.
2) Функции f: X -> Y и g: X -> Z могут быть разными даже если выполняется f(x) = g(x) для всех элементов x из множества X.
354 840782
>>40777
Хуйня твоё определение морфизма (абсолютные синонимы — мэппинга, функции).
Быстрый пример: у меня есть скалярный вход s и векторный выход
r = (cos s)e1 + (sin s)e2 + (s/10)e3
355 840783
>>40782
Линейное отображение - это не функция, это морфизм вообще в другой категории (k-Vect). Функция же - это морфизм в Set.

>морфизма (абсолютные синонимы — мэппинга, функции).


Морфизм и функция - это разные понятия. Под "мэппингом" (по-русски говорят "отображение") вообще подразумевают разные вещи.
356 840788
>>40783
В мом примере мэппинг совсем не линейный, в нём блять синус и косинус есть — где ты линейность увидел?..

И ещё раз говорю тебе: морфизм, мэппинг и функция это абсолютные синонимы, нехуй пытаться делить их. И сожги ко всем херам любые книги, где пытаются запутать и обмануть читателя тем, что пишут что функция это не одно и то же с мэппингом и морфизмом.
Screenshot from 2021-04-04 21-58-05.png37 Кб, 710x302
357 840789
358 840791
>>40789
Говорят "отображение симплициальных множеств", но не говорят "функция между симплициальными множествами". Так что морфизм = отображение, но функция - частный случай этого понятия (в категории Set).
359 840794
>>40788

>И сожги ко всем херам любые книги, где пытаются запутать и обмануть читателя тем, что пишут что функция это не одно и то же с мэппингом и морфизмом.


Придется сжечь всю литературу по теории категорий.
360 840795
>>40782

>Быстрый пример: у меня есть скалярный вход s и векторный выход


>r = (cos s)e1 + (sin s)e2 + (s/10)e3>>40788


>В мом примере мэппинг совсем не линейный, в нём блять синус и косинус есть — где ты линейность увидел?..


Действительно. Тогда это функция между множеством скаляров и множеством векторного пространства. Внезапно, вектора тоже могут быть элементами множества (ими они и являются в ZFC).
361 840799
>>40791
Ну-ка, а дискретные функции у тебя функции? На входе натуральное число n, на выходе факториал входа n!
У меня это конечно же функция, никакой ни непрерывности, ни прямой ни обратной сюръективности для бытия функцией не нужно.
А эллиптический интеграл второго рода это функция?
А бесконечный ряд Fourier?
Screenshot from 2021-04-04 23-05-18.png39 Кб, 559x757
362 840802
>>40795
Если что, вот как выглядит график функции типа >>40782
(для веселья тут эллипс вместо циркля)
363 840804
И ещё, в вычислении длины графика >>40802
E=E(L) это как раз есть эллиптический интеграл второго рода
364 840808
Для самых любителей математики - проблемка

Найти производную функции

r(s) = (2 cos s)e1 + (sin s)e2 + (s/10)e3

и рассказать её так сказать смысл
Screenshot from 2021-04-04 23-28-35.png39 Кб, 983x260
365 840811
Офигенный ролик https://www.youtube.com/watch?v=wCZ1VEmVjVo
про производную функций

(На пикче написано
производная это не наклон, не коэффициент наклона, не тангенс угла наклона графика)
366 840837
>>40808

>r(s) = (2 cos s)e1 + (sin s)e2 + (s/10)e3


>Найти производную



dr = d(2 cos s)e1 + d(sin s)e2 + d(s/10)e3
= 2 (- sin s) ds e1 + (cos s) ds e2 + (1/10) ds e3

r' = dr/ds
= ( 2 (- sin s) ds e1 + (cos s) ds e2 + (1/10) ds e3 ) / ds
= 2 (- sin s) e1 + (cos s) e2 + (1/10) e3

>рассказать её так сказать смысл


Производная тупо, а чо
367 840840
>>40777
Дело в том, что существует такая штука: протоморфизм. Её ввел Фрейд, автор теоремы имени себя. О протоморфизме можно думать как о чём-то, из чего получается морфизм путем уточнения домена и кодомена. Один и тот же протоморфизм может давать разные морфизмы при разных выборах домена и кодомена.

Подмножество f декартова произведения XxY такое, что для любого x существует единственный y такой, что пара (x, y) является элементом f, называется функциональный график, или просто график. Графики - это протоморфизмы для Set. Если взять график и специализировать его домен и кодомен, то получится как раз твоя упорядоченная тройка - то бишь морфизм в Set. Понятно, что в этом примере разными образами можно выбирать только кодомен, домен-то графиком определяется однозначно.

Во многих ситуациях разумнее изучать не морфизмы, а протоморфизмы. Именно поэтому в абстрактной теории множеств отображениями чаще называются не тройки, а просто графики, - потому что тексты про протоморфизмы для Set легковеснее текстов про морфизмы в Set. Бурбаки (у которого отображение - это именно тройка) тут, скорее, исключение.

>>40782
Первопорядковая логическая функция может не истолковываться как отображение каких-то множеств.
Далее, морфизм может быть истолкован как отображение некоторых множеств только для так называемых конкретных категорий. Можно рассмотреть (сверхбольшую) категорию, объект которой - какое-нибудь произвольное множество, а морфизмы - все множества вообще; композиция - объединение множеств. Легко видеть, что думать о морфизмах этой категории как об отображениях неразумно.
Поэтому морфизм, отображение и функция - разные понятия.

>>40783
Ну, мне кажется, что морфизмы Set корректнее называть отображениями. Функция все-таки более широкое понятие - бывают и первопорядковые функции, и ещё всякое. В большинстве случаев можно отождествлять функции и отображения, но вообще-то экстенсионал этих понятий различается.

>>40788
А давайте не будем сжигать книги.

>>40808

>рассказать её так сказать смысл


Ротор поля наподобие дивергенции градуирует себя вдоль спина и там, внутре, превращается в синекдоху бесконечно-малого.
367 840840
>>40777
Дело в том, что существует такая штука: протоморфизм. Её ввел Фрейд, автор теоремы имени себя. О протоморфизме можно думать как о чём-то, из чего получается морфизм путем уточнения домена и кодомена. Один и тот же протоморфизм может давать разные морфизмы при разных выборах домена и кодомена.

Подмножество f декартова произведения XxY такое, что для любого x существует единственный y такой, что пара (x, y) является элементом f, называется функциональный график, или просто график. Графики - это протоморфизмы для Set. Если взять график и специализировать его домен и кодомен, то получится как раз твоя упорядоченная тройка - то бишь морфизм в Set. Понятно, что в этом примере разными образами можно выбирать только кодомен, домен-то графиком определяется однозначно.

Во многих ситуациях разумнее изучать не морфизмы, а протоморфизмы. Именно поэтому в абстрактной теории множеств отображениями чаще называются не тройки, а просто графики, - потому что тексты про протоморфизмы для Set легковеснее текстов про морфизмы в Set. Бурбаки (у которого отображение - это именно тройка) тут, скорее, исключение.

>>40782
Первопорядковая логическая функция может не истолковываться как отображение каких-то множеств.
Далее, морфизм может быть истолкован как отображение некоторых множеств только для так называемых конкретных категорий. Можно рассмотреть (сверхбольшую) категорию, объект которой - какое-нибудь произвольное множество, а морфизмы - все множества вообще; композиция - объединение множеств. Легко видеть, что думать о морфизмах этой категории как об отображениях неразумно.
Поэтому морфизм, отображение и функция - разные понятия.

>>40783
Ну, мне кажется, что морфизмы Set корректнее называть отображениями. Функция все-таки более широкое понятие - бывают и первопорядковые функции, и ещё всякое. В большинстве случаев можно отождествлять функции и отображения, но вообще-то экстенсионал этих понятий различается.

>>40788
А давайте не будем сжигать книги.

>>40808

>рассказать её так сказать смысл


Ротор поля наподобие дивергенции градуирует себя вдоль спина и там, внутре, превращается в синекдоху бесконечно-малого.
368 840841
>>40837

>Производная тупо, а чо


А я вижу в ней эллипс
369 840842
>>40840

> давайте не будем сжигать книги.


А чем зимой греться, чтобы не сдохнуть в этой непригодной для жизни человека местности?
370 840845
>>40842
Бугуртом.
371 840849
>>40837
Это чем-то отличается от школьной производной, которой ломают и мучают школьников?
372 840850
>>40849
Это то же самое, просто в универе дают более формальное определение и находят производные более сложных функций
373 840862
>>40779

>могут быть чем угодно - не обязательно числами.



Это уже функционал будет, а не функция, не?
374 840864
>>40862

In mathematics, the term functional (as a noun) has at least three meanings.

• In modern linear algebra, it refers to a linear mapping from a vector space V into its field of scalars, i.e., it refers to an element of the dual space V∗.

• In mathematical analysis, more classically and historically, it refers to a mapping from a space X into the real numbers, or sometimes into the complex numbers, for the purpose of establishing a calculus-like structure on X. Depending on the author, such mappings may or may not be assumed to be linear, or to be defined on the whole space X.

• In computer science, it is synonymous with higher-order functions, i.e. mappings that take mappings as arguments or return them.
375 841051
>>40811
Да, забавный ролик.
376 841227
>>23017 (OP)
Как смотрите на использование Aluffi Algebra: Chapter 0 для повторения абстрактной и линейной алгебры? Или есть варианты лучше?
377 841372
Насколько хорошо в Винберге линейная алгебра изложена? Стоит ли ее по нему изучать? Чистым математиком быть не собираюсь, мне для теоретической физики
378 841403
>>41372
Винберг тебе не пригодится.
379 841563
>>41227
На мой взгляд, Алуффи плохо написан, особенно поздние главы.
По линейной алгебры лучший вариант - Blyth Module theory (написан очень понятно, не требует знакомства с линейной алгеброй; ошибки и неточность есть, но не больше чем в других учебниках)
По абстрактной алгебре советую Grillet - Abstract algebra
380 841583
>>23017 (OP)
Начну с начала - в школе не шарил в математике от слова совсем. Были конечно всякие походы к репетитору, которые научил меня дескриминантам и прочему, но по сути я научился только трюкам.

Потом были попытки в мат.анализ и абстрактную алгебру ни то ни другое я не осилил. Так же была попытка ворваться в дискретную математику и теорию конечных отоматос.

После длительного перерыва попыток вката решил возобновить занятия, но не знаю с чего начать и чего хочу. В планах есть ковырять потихонечку CPL и попытаться стать байтослесарем и писать что-нибудь архаичное.

Думаю там бы теория конечных отоматос пригодилась бы. Так же хотелось бы начать шарить в абстрактной алгебре, теория колец, теория полей, гомоморфизмы.

Вопрос - с чего начать, куда идти и что делать? Вобщем нужен примерный роадмап от шарящих со списком книг и т.д. Буду рад советам.
381 841585
>>40613
Что значит концептуально?
382 841586
>>41583
Зачем тебе это? Большую часть математики без универа не изучить
383 841588
>>41586
Ты вадик?
384 841593
>>41586
Короче жду советов от невадиков.
385 841598
>>41583
>>41583
Математика – не та наука, которую можно изучить самостоятельно.
386 841599
>>41598
Если есть хорошие учебники, задачники и мотивация - можно.
387 841603
>>41598
Вадик, просто игнорируй это сообщение. Буду ждать советов от нормальных анонов.
388 841609
>>41583

>CPL



что это?

>абстрактной алгебре, теория колец, теория полей, гомоморфизмы.



Гомоморфизмы как-то выбиваются из ряда. Ощущение, что ты просто слов накидал которые круто звучат с твоей точки зрения. Ну с Винберга начни что ли.
389 841613
>>41609

>что это?


C Programming Language

>Гомоморфизмы как-то выбиваются из ряда.


Я чому-то думал, что гомоморфизмы это как раз из абстрактной алгебры.
390 841637
>>41613
1)нахуя тебе си?
2)нахуя тебе абстрактная алгебра?
391 841638
>>41637
Просто я безработный. Поэтому пока я безработен я решил, что неплохо было бы занять себя чем-то развивающим.
392 841666
>>41613
Гомоморфизм - это такая функция, у которой входы и выходы одного типа (на входе псевдовектор и на выходе псевдовектор, на входе скаляр и на выходе скаляр - в школах проходят вот только такие гомоморфизмы)
393 841667
>>41613
Ну я к тому, что абстрактная алгебра (она же высшая, но видимо слово "высшая" сейчас недостаточно круто звучит) - это раздел математики, теория колец и теория полей - разделы абстрактной алгебры, а гомоморфизм это понятие из алгебры и если ты начнешь изучать теорию колец и полей, то обязательно столкнешься с гомоморфизмами. Нахуя ты отдельно выписал, что хочешь заниматься гомоморфизмами - непонятно. Выглядит как набор buzzwordов, чтобы тяночки текли.
394 841669
>>41666
>>41667
Я просто помню, что встречал этот термин в учебниках, поэтому и его написал.
395 841691
Почему топологию называют центром математики? По моему центр математики это алгебра!
396 841760
>>41609
Винберг - плохой учебник. В нём нет доказательств, в нём только идеи; восстанавливать доказательства целиком предлагается читателю самостоятельно.

Нормальные учебники - это Dummit, Foote. Abstract algebra либо Grillet. Abstract algebra.
397 841961
>>23386
Возможно, что подразумевалось, что ты будешь решать этот пример через логарифмирование. От этого появляется ещё доп. ограничение.
398 841970
>>41760

>восстанавливать доказательства целиком предлагается читателю самостоятельно


Ну так он и не писался для тупых и ленивых.
399 841974
>>41760
>>41970
Так что посоветуете? Можете дать roadmap? Типа Фихтенгольц ー>Винберг и так далее. Хотел бы заняться именно абстрактной алгеброй. Или дискреткой на худой конец.
Screenshot12.png73 Кб, 818x206
400 841977
>>23017 (OP)
Почему у нас есть такое право? Поясните пожалуйста.
401 841979
>>41977
Если она сократима - будем сокращать, сокращать и сокращать, пока не станет несократимой.
402 841981
>>41974
Шапку не пробовал читать? http://verbit.ru/Job/HSE/Curriculum/all.txt
403 841987
>>41979
Сэнкс.
>>41981
Мне сказали что вербицкий и сам этого не читал, с чего я должен да и к тому же у меня этот сайт не открывается,? Я скачал Фихтенгольца, потому что в треде Зорича назвали плохим. Но хочу заниматься абстрактной алгеброй, но мне роадмапа не дали, поэтому я сижу тут и задаю вопросы.
404 841995
>>41987
Вербицкий это читал. Более того, Вербицкий всё это отпреподавал. Это его вторая программа, специально для студентов-самоучек.

Скопировала на pastebin - думаю, уж он-то откроется.
https://pastebin.com/raw/6ajpQ7J5
405 841997
>>41987

>потому что в треде Зорича назвали плохим


Они уже давно забрали свои слова обратно. А я забрал их зубы и пока не планирую возвращать.
406 842002
>>41987

>потому что в треде Зорича назвали плохим


Теперь посчитай, сколько раз Зорича назвали хорошим. Не только на этой доске, а вообще в целом. Думаю, вывод ты сделаешь. Миллион довольных покупателей и один злобный хейтер на дваче. Интересно, кто прав?
407 842007
>>41987
Зачем тебе Фихтенгольц если ты хочешь изучать алгебру? Хотя Фихтенгольц классный. нет я не зоричешизик Но, наверно, к современной математике уже мало относится. Вон тебе ОП кинул 2 книжки, ебашь.
408 842046
>>42007

>Фихтенгольц классный


У меня от тебя варианта.
409 842047
>>41995
Сэнкс. Но вот только вопрос - реально ли это изучить самому в 29 лет? И вообще тут кто-либо осилил т.н. core math или вы тут дружно троллите меня?
>>42007
Где? Я что-то не вижу...
410 842049
>>42047
Никто кроме тебя не может сказать, что для тебя реально, а что нет.
411 842054
>>41979
А q там нечётное, потому что p чётное и тогда дробь (p/q)^2 от постоянного сокращения становится равной 0.5? Только не смейтесь надо мной.
412 842055
>>42054
А нет, не то.
413 842056
>>42054
>>42055
Я кажется понял - q нечётное т.к. дробь нескоратимая. Жалко что Фихтенгольца мне читать не нужно. Пойду купить core math.
414 842062
>>23127
уже
415 842066
>>23116
Значит ли это что вперед матана и других областей надо изучить теорию множеств?
416 842070
>>42054
Дробь несократимая. p четное. Значит, q нечетное, - иначе бы дробь была сократимой.

>>42066
В любом учебнике анализа всё необходимое из теории множеств вводится в первой главе (или в приложении). Отдельно что-то читать, скорее всего, не нужно.
417 842072
>>42046
Ну 1 устаревший термин, объясняет-то классно.

>>42047

>Где? Я что-то не вижу...



Вот же

>Нормальные учебники - это Dummit, Foote. Abstract algebra либо Grillet. Abstract algebra.



>Но вот только вопрос - реально ли это изучить самому в 29 лет?



Лично мое мнение, что нет. Те, кто могут, демонстрируют свой талант в раннем возрасте. Обычным людям надо хотя бы 2 курса матфака походить, чтобы понять как заниматься, как читать книги, на что обращать внимание, получить какие-то основы мат.культуры.
418 842081
>>42072
Тогда наверное лучше изучать алгебру и матан и не тратить время на core math?
419 842087
>>42081
Да мы ж не знаем какие у тебя цели. Изучай, то что нравится.
420 842365
>>23017 (OP)
Какие области математики нужно изучить что бы работать в области ИИ?
421 842373
>>42365
Линал, матстат, комбинаторика, теория графов. В серьезных вещах может понадобиться функан.
422 842578
Как это учить-то? Вот я скажем так скачал учебник, читаю его. Там теоремы, аксиомы, я их понимаю, далее свойства какие-то из этого всего, и я скажем так к этому моменту могу уже забыть что там было, возвращаюсь по новой. Как правильно учить? Конспекты писать по каждому учебнику? Если так, то оно конечно немного поможет с запоминанием, но не исключит эту проблему вообще, всё равно придётся по 10 раз возвращаться к одному и тому-же + больше времени будет уходить.
423 842586
>>42578
а задачи ты решаешь?
424 842605
>>42578
Начал читать книгу по грамматике восточных языков и понял, что читать такую литературу я не умею. В итоге периодически перечитываю параграфы не надеясь, что я что-то запомню и пойму с первого раза, но надеюсь что в перспективе я запомню и пойму.

Ну и непонятные моменты я выписываю, что бы потом погуглить вне книги. Это конечно не математика, но я хочу попробовать такой метод на математике.

Вообще споймал себя на мысли, что читая любую книгу я на самом деле просто узнаю слова и буквы не вчитываясь в контекст и смысл книги, как будто где-то витаю. Надо с этим что-то делать.
425 842621
>>42605
Ну я как бы понимаю, но когда в книге в последствии могут быть сноски на теорему, я теряю суть. Тобишь ты возвращаешься опять назад, а там выясняется что нужно опять проетать несколько глав чтобы в это старое вьехать. Допустим тот же Зорич. Или это особенности книги?
426 842622
>>42586
Не всегда. Я в шапке, к примеру, видел много разных книжек в основном это учебники, реже пособия, а задачников нет вроде.
427 842823
Студенты-математики, как вы относитесь к теории множеств и категорий? Вас не смущает всё это? Может математика не познаваема? По крайней мере человеком.
44321E99-D46D-49E2-93CB-6131FA68E0E0.jpeg163 Кб, 750x588
428 842863
429 842866
>>42823
А кем, если не человеком?
430 842868
>>42866
Каким-нибудь йоба интеллектом. Люди же не пытаются бежать со скоростью 1000 км в час - используют самолёт.
431 842894
>>42868
Но в случае бега ты точно знаешь, что быстрее 45 км в час ты не можешь пробежать. А в случае математики (мозга) мы (пока) такого ограничения не знаем. Так что Работаем дальше
432 842895
>>42894
Ничего не получится...
433 842900
434 842903
>>42900
Не верь мне. Убедишься сам. Или беги оттуда. Беги из этой пропасти...
11141632035dcd02f3c22a4.jpg70 Кб, 500x620
435 842958
>>42823
Нормально относимся. Готовимся к EGA.
436 843021
>>23017 (OP)
Я наверное уже всех достал, но задам последний вопрос - что надо пройти перед изучением абстрактной алгебры, что бы потом заниматься только абстрактной алгеброй и заниматься проблемсолвингом только в этой области?
boocover.jpg19 Кб, 200x305
437 843061
>>43021
Достаточно наивной теории множеств в объеме книжки Хаусдорфа. Можно, конечно, и по другим книжкам, а из этой взять просто список тем.
438 843125
>>43061
Благодарю, анон.
Безымянный.jpg102 Кб, 1073x640
439 843151
>>42578
Учишь английский. На английском на либгене дохуище учебников с решебниками по всем разделам математики, кроме самых продвинутых ( у меня у самого 6 Гб таких учебников с решебниками на диске). Читаешь главу, решаешь задачи (если их много, то решаешь только четные номера задач). Если ездишь в общественном транспорте, там можешь читать книгу по изучаемому тобой предмету на русском (как вспомогательную литературу).

Как искать:
1) Сначала ищем решебник: заходим на libgen.rs, вводим название предмета на английском и слово solution, выбираем Search with mask (word*): Yes и нажимаем поиск.
2) По найденным решебникам ищем оригинальные учебники (есть в 99 % случаев) по названию предмета и фамилии авторов. На Амазоне можно посмотреть отзывы, если что.
3) Занимаемся.

>>43021
Линейную алгебру.
440 843153
>>42605
А занимался бы по учебникам с упражнениями и ключами в конце учебника (выполняя все эти упражнения) - все бы запоминал.
441 843157
>>42072

>Dummit, Foote. Abstract algebra


Это скорее хардкорный справочник.

Если брать именно учебники, то есть два таких варианта с решебниками:
http://libgen.rs/book/index.php?md5=C5CF8FC3A942610ED586BB708E53C937
http://libgen.rs/book/index.php?md5=7C7DD82AAE364B55EBDB70B8971D40F2

http://libgen.rs/book/index.php?md5=D11F196333536846C63B9D7CD6C59A45
http://libgen.rs/book/index.php?md5=3F7C108022503CE7A292065197A6EB7D
442 843158
>>43151
Спасибо.
443 843161
>>43158
Ты тот, кто спрашивал за введение в высшую алгебру? Английский понимаешь?
444 843162
>>43161
Я тута. Английский понимаю на разговорном уровне. Сейчас учу японский. За совет благодарю.
445 843163
>>43161
Да, у меня если что гугл переводчик есть. А так понимаю немного. Да и учебник, это ведь не художественная литература.
446 843168
>>43162
Ну раз понимаешь, проходи в самом начале вот это (с решением задач)
http://libgen.rs/book/index.php?md5=B67298CBE7B0B63977F4845D0258E92B
http://libgen.rs/book/index.php?md5=C0A8763821C726725E4CCABDFEAF7859
По сути, это реально единственный пререквизит, там есть и теория множеств, и логика, и введение в линейную алгебру, теорию групп и колец. После этого бери какой-то из этих учебников и решай >>43157

Если решишься еще взять отдельный курс по линейной алгебре, то как нацеленному на чистую математику советую Ленга:
http://libgen.rs/book/index.php?md5=76FFC3E1143750908F207E36B8755E38
http://libgen.rs/book/index.php?md5=422E04699CFA5B3DFBAB2B47554A569F
447 843172
>>43168
Сенкс анон! Буду смотреть, читать.
448 843175
>>43172
Ну вот, а уже после прохождения какого-то из этих учебников >>43157 можно будет брать серьезный повторный курс как-то
Ленга
http://libgen.rs/book/index.php?md5=797BD250B8D7B352049C7C99DA308912
Ротмана
http://libgen.rs/book/index.php?md5=84FC664D285A70F3E6AB93C6A9FDE5EA
http://libgen.rs/book/index.php?md5=83128A4F9B0056E55B67C30B710294B9
Хангерфорда
http://libgen.rs/book/index.php?md5=0887DE6DA1CACC32A788AB3E22CC559A
Биркоффа-Маклена
http://libgen.rs/book/index.php?md5=DBAB3FD0A2899A474A0B386296E5FC59
Даммита-Футе, Алуффи и т.д.

Да даже это, если ты какой-то там погромист
http://libgen.rs/book/index.php?md5=176C3CFD768173A1D04BFB6D99D6341B
449 843181
>>41583
Чувак, тебе нужна дискретная математика для начала, а не высшая алгебра.
Бери какую-то из этих трех книг (которая понравится по обсуждаемым темам), да занимайся.
http://libgen.rs/book/index.php?md5=26DE073344D4C6172C8FE43E86A5CA53
http://libgen.rs/book/index.php?md5=B6D85648C6BC0D9028267AADB14D3267

http://libgen.rs/book/index.php?md5=E1AE9ADAC5F04BFCF45826428B68AA78
http://libgen.rs/book/index.php?md5=C906B91C2718096BA2E704243FFDEDA8

http://libgen.rs/book/index.php?md5=66D70775F0529CFCDBA7E2FE021A2C9E
http://libgen.rs/book/index.php?md5=618756E719E24EF9BBDE66B7C2BEEE3D
http://libgen.rs/book/index.php?md5=27D13AD5BFD200A8E72D9596743BB122

Если ты и школьную плохо помнишь, то вообще надо начинать с прекалькулюса. Да и линейная алгебра, как для погромиста, для тебя будет полезней, чем высшая.
450 843182
>>40571
Как первый курс: Ленг, потому что есть решебник.
http://libgen.rs/book/index.php?md5=B56871DEA22A48AEBFCC2096D1A40223
http://libgen.rs/book/index.php?md5=FB66B27E29840C83269B0F9FC11EFCA1

Без решения задач вообще похуй что читать. В таком случае можешь новый пятитомник Барри Саймона навернуть на (в сумме) 3000+ страниц.
http://libgen.rs/search.php?req=barry+simon+Comprehensive+Course+in+Analysis&open=0&res=25&view=simple&phrase=1&column=def
451 843184
>>40590
Мало вводных. У американцев есть книги по этим предметам как для вчерашних школьников, так и для продвинутых юзеров. Если ты собираешься просто их читать без выполнения упражнений, можешь брать серию "Курс высшей математики и математической физики" и почитывать для начала, она вполне годная для прикладников и интересующихся.
452 843185
>>43175
Ого! Ленга уже скачал, но пока не смотрел. Их все читать или можно кого-то одного выбрать?
453 843186
>>40590
А, ну и тащемта, есть такой список:
http://4chan-science.wikia.com/wiki/Mathematics

Предлагаю закрепить его в шапке.
454 843187
>>43185
Не, кого-то одного. Двухтомник Ротмана - самый объемный. Можешь попробовать потом его, если понравится стиль - у него есть вводные книги и по другим разделам алгебры (An Introduction to Homological Algebra, An Introduction to Algebraic Topology, An Introduction to the Theory of Groups).
455 843189
>>43185
Если же ты просто хочешь почитать что-то подготовительное перед сном, пока проходишь Mathematical Proofs. A Transition to Advanced Mathematics, то можешь попробовать это
http://libgen.rs/book/index.php?md5=1636F492B4244B37D264E7FC9A0799A7
http://libgen.rs/book/index.php?md5=78CC46CB3DD667AD263367981591C6BA
456 843190
>>43189
Добавлю, что второе издание Пинтера в лучшем качестве тяжело найти.
Либо читать epub
http://libgen.rs/book/index.php?md5=653779D383E51226AA47D9714AA60C8F
Либо первое издание
http://libgen.rs/book/index.php?md5=741D7289B60599A0AB489E4108233DBB
Безымянный.jpg151 Кб, 1035x276
457 843195
>>23243
>>23246
По анализу - пикрил, качайте с либгена
458 843196
1
459 843426
>>43175
Привет! Вопрос такой, если я знаю материал учебников отсюда >>43157 , как именно надо браться за upperundergrad-учебники типа Ротмана, Рудина? Проблема в том, что среди таких крутых учебников решения есть только для Дамми-Фута и Рудина, и мне не совсем понятно, смогу ли я выучиться без solutions manual. Ну типа все самому без подсказок на этом уровне уже немного вредновато делать... Есть мысли какие-то на этот счет? Может стоит комбинировать: читать один учебник(условно Аман-Ашер/ Ротман), а упражняшки делать из Рудина/Дамми-Фута, имея возможность проверки, например?
460 843476
>>43426

>Вопрос такой, если я знаю материал учебников отсюда


Если ты не можешь решить задачи из этих учебников, значит материал ты оттуда не знаешь. Все просто. Кстати, ты почему-то мешаешь учебники по анализу Рудина/Аманна-Эшера с высшей алгеброй. Если изучаешь анализ, начинай с учебника Ленга>>43182, у него также есть учебник по комплексному анализу с решебником.
http://libgen.rs/book/index.php?md5=D37B0F7B8EA860CD1CA63DFBADD726FF
http://libgen.rs/book/index.php?md5=6071FB476CBA371A2988B5DB5AD949CC

По вещественному анализу есть такое:
http://libgen.rs/book/index.php?md5=2075275DA06579AEF304F65FC3387503
http://libgen.rs/book/index.php?md5=529127DF6F6FBD70FE5B746AA43B8082

http://libgen.rs/book/index.php?md5=19BC4D740CDCA0186C14A5521BD7431F
http://libgen.rs/book/index.php?md5=DB166CE46AD26F7CEF5B67C288E2259E

http://libgen.rs/book/index.php?md5=DC0BBDA31199EC2B16ACB82CD6AC762D
http://libgen.rs/book/index.php?md5=F1E08D550360E26841EB50199BDD71CF

http://libgen.rs/book/index.php?md5=E2D11340469D27614F4B0025CD466CAA
http://libgen.rs/book/index.php?md5=7282867911149F94C1E996A3BA769CED

>>43426

>Может стоит комбинировать


Попробуй, только чтение книги, решебника к которой нету, ты должен отодвинуть на второй план: читать в общественном транспорте или перед сном. У меня, например, если отличная коллекция совковых учебников (физических копий), я их читаю в метро. Интересные для меня англо- или немецкоязычные книги без решебников я читаю перед сном. А уже за компом я читаю и занимаюсь по книгам с решебником.
Только учитывай то, что я - НИИшник-металлофизик, всю необходимую для меня математику я знаю, а так математикой занимаюсь чисто как хобби и для расширения областей знания, времени у меня полно и я никуда не спешу, поэтому может мой подход - не совсем оптимальный с точки зрения тайм-менеджмента (стараюсь решать все упражнения, поэтому на это также уходит много времени).
461 843506
>>43476
Анон, расскажи что-нибудь про металлофизику. Как учился, какая математика нужна и так далее.

Я сам сейчас на физика учусь, хоть и планирую в гораздо более теоретическую область пойти космология, лол, но всё равно интересно.
462 843556
>>43506
Та рассказывать особо нечего, в пост-совковой науке сейчас не так уж и интересно. Если думаешь идти в науку - рекомендую уже сейчас начинать дрочить какой-то язык программирования (как минимум Python, как максимум - C/C++): будет тебе подспорьем в научных вычисления, а если что - перекатишься куда-то в другую область.
У нас более прикладной отдел, поэтому я больше тяготею к штукам типа теории упругости, термодинамики и металловедения, нежели к физике твердого тела, кристаллографии и прочей электронной теории металлов. Если говорить о математике - то нужен стандартный набор физика/инженера, можешь посмотреть список топиков в каком-то курсе высшей математики (типа Смирнова) или в англоязычных учебниках-справочниках (например, "Mahematical methods for physics and engineering" Riley, Hobson, Bence). Вкратце - первая ступень:
1. Дифференциальное и интегральное исчисление
2. Линейная алгебра + теория матриц
3. Векторное и тензорное исчисление
4. Анализ (вещественный, опционально - комплексный)
4. Обыкновенные дифф. уравнения + уравнения математической физики (с преобразованием Лапласа/Фурье и рядами Фурье)
5. Теория функций комплексного переменного
6. Вариационное исчисление
7. Теория групп
8. Опционально - интегральные уравнения, теория специальных функций, вычислительная математика.
9. Если ты физик-экспериментатор - желательно учить теорвер, матстат и планирование эксперимента.

Вторая ступень:
1. Функциональный анализ, опционально - гармонический.
2. Топология и дифференциальная геометрия (многообразия, дифференциальные формы, расслоения, характеристические классы, риманова геометрия и т.д.).
3. Алгебраическая топология.
4. Группы и алгебры Ли; теория представлений.
463 843575
2
464 843694
>>43476

>Если ты не можешь решить задачи из этих учебников, значит материал ты оттуда не знаешь.


В первом издании Лэнга была следующая задача: "Возьмите учебник по гомологической алгебре и докажите все теоремы самостоятельно". Вряд ли кто-то это сделал. Видимо, никто из читавших Лэнга не знает базовой алгебры.
465 844021
Решил сдавать профильный матан для подстраховки, если все же не получится поступить куда запланировано (гумфак)
По математике полный ноль, думать не умею, задачи решать — тем более. Разве что по алгебре могу алгоритм запомнить и как-то на уровне школьном, а учусь я в обычной школебазу-базу прям решить. Допускаю ошибки в арифметике и в целом не понимаю как устроено всё.
Посоветуйте учебники, с чего начать? Что делать потом и как ботать задания, если я совсем не умею в логику
Сдаю егэ через год, если что. Рассчитываю на баллов 70 (?)
466 844023
Решил сдавать профильный матан для подстраховки, если все же не получится поступить куда запланировано (гумфак)
По математике полный ноль, думать не умею, задачи решать — тем более. Разве что по алгебре могу алгоритм запомнить и как-то на уровне школьном, а учусь я в обычной школебазу-базу прям решить. Допускаю ошибки в арифметике и в целом не понимаю как устроено всё.
Посоветуйте учебники, с чего начать? Что делать потом и как ботать задания, если я совсем не умею в логику
Сдаю егэ через год, если что. Рассчитываю на баллов 70 (?)
467 844024
>>44023
Ёбаная ошибка постинга!
468 844056
>>44023
Гельфанд "Алгебра"
469 844152
>>44023
Алгебра 7,8,9 класс
470 844169
>>44056
Спасибо!
>>44023
Разве этого хватит для профильного матана?
471 844170
>>44169
Порешай алгебру 7-9(алгебра Шеня хороша, но она маленькая , и задач там немного, но тебе может и этого хватить), что-бы много времени не терять, делай только номера с ответами(чётные как правило), сначала решаешь парочку первых номеров, если легко идёт - пробуешь решить последние , если не получается - возвращаешься и прорешиваешь середину.
Решать задачи обязательно, иначе ничему не научишься.
После этого открываешь учебник по алгебре 10-11, смотришь главы про тригонометрию, теорию вероятности и производные
Учебники 7-9 - тебе стоит освоить и прорешать за месяца полтора-два
Месяц на 10-11, потом просто найдёшь какой-нибудь курс подготовки к егэ, по типу Трушина, и там тебе уже объяснят всё что нужно, посмотришь как другие решают задачи
472 844243
>>44023
https://www.youtube.com/watch?v=qlZMNryduHU
Канал "Математик МГУ".
Мужик готовит к ЕГЭ и ДВИ МГУ, так что у неё много годноты на канале.
Сам найдешь у него список рекомендованной литературы.

А если честно, то не занимайся самообманом и найми себе репетитора, в одиночку математика не учится.
473 844245
Бамп
474 844263
>>44170
Спасибо тебе большое, анонче :3
>>44243
Спасибо! Репетитор был одно время, но я его дропнул, тк чувствовал, что мне не хватает теории и я учусь только по тупому запоминать алгоритмы решений, но не понимаю откуда все вытекает на самом деле. Сейчас думаю для начала самому теорию изучить, попрактиковаться немного, а потом уже, когда более свободно буду разбираться и вопросы начнут возникать четко сформулированные, тогда и найму скорее всего повторно.
475 845416
>>44023
Хз что там сейчас в ЕГЭ, при мне такой хуйни не было. Но если тебе нужен полноценный дистиллированный школьный курс на русском, бери Сканави
Сам учебник:
http://libgen.rs/book/index.php?md5=9B9E43387F2FE19AF1495A11D78565BC

Задачник:
http://libgen.rs/book/index.php?md5=9D380602EBEE5C0AD5DFCBF53D5075DF

Решебники к задачнику:
http://libgen.rs/book/index.php?md5=9166930CDEB5D8600DC8DCF60EC5B871
http://libgen.rs/book/index.php?md5=1B8232E3144461777B4A8B81BD6AB4D5

Не знаю, надо ли вам там сейчас интегралы и пределы, но можешь потом посмотреть этот двухтомник:
http://libgen.rs/book/index.php?md5=75ECFF63FC974D7E04A9CD070C65F257
http://libgen.rs/book/index.php?md5=BED423D96A840A3DF7E4EED564651900

Сделаем из тебя математика, короче.
intelgold.png68 Кб, 550x550
476 845441
>>45416
Спасибо тыщу золотых зеонов в благодарность
477 845442
>>44023
ЕГЭ лучше Сканави
478 845928
>>45416
Спасибочки :3 Уже подумываю сделать матан приоритетным предметом.. Но боюсь обосраться.
мимо >>>844021 анон
479 845940
>>45416
Большое спасибо! Ты и мне очень помог.

Очень сильно извиняюсь за оффтоп, но нет ли у тебя подобных материалов по школьной физике? Или можешь посоветовать место для такого вопроса?
480 845951
>>45940
Яковлев.
481 846297
>>45940
Если мы говорим про школьников, то по теории Ландсберга можешь почитать
https://rutracker.org/forum/viewtopic.php?t=4386654
Просмотри комментарии в теме, чтобы лучше ориентироваться (может тебе лучше взять стандартный учебник, но я не знаю, какие там школьные учебники сейчас считаются хорошими). Тамошний спор об эфире можешь пропустить.

Задачник уж подбирай сам - скачай все и посмотри, какой из них тебе больше подходит.
Принципы решения задач
https://rutracker.org/forum/viewtopic.php?t=5235631
Задачник
https://rutracker.org/forum/viewtopic.php?t=4191608
Решебник к нему
http://libgen.rs/book/index.php?md5=04D4EF959E551F095578E331BAE3F8F4

Есть еще такой вариант:
10 класс
https://rutracker.org/forum/viewtopic.php?t=2993621
https://rutracker.org/forum/viewtopic.php?t=3912015
http://libgen.rs/book/index.php?md5=1F49069DECB7602E283F02E8F8E3E8C1
11 класс
https://rutracker.org/forum/viewtopic.php?t=3254680
https://rutracker.org/forum/viewtopic.php?t=3912145
На либгене можно найти более новые учебники и сборник задач из этой серии, но вот решебники есть только к старому изданию 2009-2011 годов.

Дополнительно
http://libgen.rs/book/index.php?md5=4750795A2B08654CCE073CB11738BF92
http://libgen.rs/book/index.php?md5=660174F19CB29304FA613B67100ED3FE
http://libgen.rs/book/index.php?md5=FE2A4FA6575AEF799D7642C959453A7C
https://rutracker.org/forum/viewtopic.php?t=3552845

Есть еще такая ЕГЭшная шняга
http://libgen.rs/book/index.php?md5=014DE041CCE3E0BA8F726669FC5BB09D
https://rutracker.org/forum/viewtopic.php?t=4932010
481 846297
>>45940
Если мы говорим про школьников, то по теории Ландсберга можешь почитать
https://rutracker.org/forum/viewtopic.php?t=4386654
Просмотри комментарии в теме, чтобы лучше ориентироваться (может тебе лучше взять стандартный учебник, но я не знаю, какие там школьные учебники сейчас считаются хорошими). Тамошний спор об эфире можешь пропустить.

Задачник уж подбирай сам - скачай все и посмотри, какой из них тебе больше подходит.
Принципы решения задач
https://rutracker.org/forum/viewtopic.php?t=5235631
Задачник
https://rutracker.org/forum/viewtopic.php?t=4191608
Решебник к нему
http://libgen.rs/book/index.php?md5=04D4EF959E551F095578E331BAE3F8F4

Есть еще такой вариант:
10 класс
https://rutracker.org/forum/viewtopic.php?t=2993621
https://rutracker.org/forum/viewtopic.php?t=3912015
http://libgen.rs/book/index.php?md5=1F49069DECB7602E283F02E8F8E3E8C1
11 класс
https://rutracker.org/forum/viewtopic.php?t=3254680
https://rutracker.org/forum/viewtopic.php?t=3912145
На либгене можно найти более новые учебники и сборник задач из этой серии, но вот решебники есть только к старому изданию 2009-2011 годов.

Дополнительно
http://libgen.rs/book/index.php?md5=4750795A2B08654CCE073CB11738BF92
http://libgen.rs/book/index.php?md5=660174F19CB29304FA613B67100ED3FE
http://libgen.rs/book/index.php?md5=FE2A4FA6575AEF799D7642C959453A7C
https://rutracker.org/forum/viewtopic.php?t=3552845

Есть еще такая ЕГЭшная шняга
http://libgen.rs/book/index.php?md5=014DE041CCE3E0BA8F726669FC5BB09D
https://rutracker.org/forum/viewtopic.php?t=4932010
482 846299
>>45940
Забей на весь этот бред. Просто смотри Пенкина.
изображение.png128 Кб, 1304x811
483 846311
Кто-нибудь хочет заниматься вместе на Khan Academy?
Там есть математика со 2 класса и минимум первый курс колледжа США. Не знаю как точно переводить их систему образования на нашу.

Я сам сейчас примерно на уровне Algebra 1/2. Это где-то старшие классы российской школы.
484 846313
>>46311
Нахуя? Бери учебник Precalculus с решебником (Стюарта, Ларсона или Экслера) да проходи. Если так уж хочется что-нибудь глянуть - смотри TTCшные лекции. Например:
https://thepiratebay10.org/torrent/7604658/TTC_-_Mastering_the_Fundamentals_of_Mathematics
https://rutracker.org/forum/viewtopic.php?t=3684121
https://rutracker.org/forum/viewtopic.php?t=3682501
https://rutracker.org/forum/viewtopic.php?t=4077299
Там дальше и по исчислению есть, и т.д.
https://www.thegreatcourses.com/category/mathematics?CFM=mega_menu
Качать или на рутрекере, или на пиратбее.

У них много всяких лекций и по музыке, и по истории, и по философии, и т.д., все - от профессоров американских универов в охуительном качестве. Вбивай на трекерах "TTC" да смотри что есть.
485 846314
>>46311

>Кто-нибудь хочет заниматься вместе на Khan Academy?


Одновременно врубать видео и по дискорду разговаривать «на уроках»? Или что...
486 846315
>>46313
В чём преимущество твоих лекций над академией кхана?
487 846316
>>46314
Как вариант.
>>46313
На Khan Academy полно видео по темам. +автоматическая система проверки заданий, надо только ответы вводить.
488 846318
>>46315
У лекций нет никаких преимуществ. Любые лекции - это обзорный материал, который должен в доступной форме дать тебе представление о предмете. Если ты действительно хочешь знать предмет - ты должен читать учебники и делать упражнения. Это единственная проверка того, что ты можешь применить полученные знания на практике.
35161A9C-3A09-4C4A-9996-EFD2C38E9582.jpeg88 Кб, 600x799
489 846319
>>46313

>Precalculus с решебником (Стюарта, Ларсона или Экслера)


>все - от профессоров американских универов

490 846321
>>46319
Школьник-политачер, не могущий в английский - ты? Расскажи что-то про хохлов.
491 846325
>>46321
Зачем ему знать английский, если он живёт в стране с топовой математической школой? В России для познания "прекалькулуса" можно просто почитать методички Гельфанда или посмотреть курс фоксфорда за пятый класс.
А хохлам-дегенератам приходится страдать и поглощать гигантские американские учебники на 5000 страниц, в которых авторы разжёвывают базовые вещи для ниггеров (которыми хохлы безусловно и являются). Зато могут пофлексить перед другими деревенщинами знанием английского на B1 и тем, что они имеют возможность читать настоящий учебник аж из америки
492 846338
>>46325

>Зачем ему знать английский, если он живёт в стране с топовой математической школой?


Затем, что среднестатистический российский школьник, судя по треду, нихуя не вывезет "топовую математическую школу", дропнет математику и в итоге станет глуповатым ура-патриотом, рассказывающим на двачах о величии Россиюшки.
А вот Прекалькулюсы и прочие учебники для высшей школы/колледжей в США написаны и отработаны так, что даже глуповатый школьник их поймет, решит задачи в достаточном количестве (а не 100500 штук, как в обычных задачниках) с возможностью перепроверки решебником, и сумеет в конце-концов полюбить математику, увидеть ее красоту на доступном для себя уровне.
493 846348
>>46338

>среднестатистический российский школьник, судя по треду, нихуя не вывезет "топовую математическую школу", дропнет математику и в итоге станет глуповатым ура-патриотом,


Человек, который осиливает математику, наоборот начинает любить Россию (как Савватеев например).

> вот Прекалькулюсы и прочие учебники для высшей школы/колледжей в США написаны


Они написаны для дебилов. Но проблема в том, что дебил не будет сам читать столько страниц занудной тягомотины. Это тяжело даже для усидчивого человека со стабильной психикой. И в конце этого "путешествия" математика будет вызывать только отвращение.

>отработаны так


Вся отработка прекалькулуса происходит на уроках математики, где училка всё объясняет. Многостраничный талмуд максимум как справочник используется. Посмотреть одну главу, если не понял училку. Ну и решить домашку.

>сумеет в конце-концов полюбить математику, увидеть ее красоту


Красоту в квадратных уравнениях и графиках функций? Да, ведь там можно сердечко отобразить... Или график синуса. Кайф. Или например пикрил. Пальцем верхнюю часть закрыл и получил i love you. Очень красиво :3
494 846351
>>46348

>Но проблема в том, что дебил не будет сам читать столько страниц занудной тягомотины.


Проблемы дебилов учебники не ебут. Если тебе вообще нудно его читать - ты его читать не будешь, даже если он написан в согласии с "топовой математической школой". А вот если человек заинтересован - он его читать будет.

>Вся отработка прекалькулуса происходит на уроках математики, где училка всё объясняет.


Тогда этот тред не нужен, закрывай. Все проходится на уроках/лекциях/семинарах, хуле вы тут вообще делаете?

>Красоту в квадратных уравнениях и графиках функций?


Для меня красота выражается в возможности прийти к одним и тем же результатам различными способами и даже с помощью различных разделов математики. Ну и теоретическая физика, конечно, возможность вывести законы природы с помощью теоретических построений, без эксперимента.
495 846358
>>46351
А ты кто?
496 846362
>>46358

Я >>43476-анон. Ну и вообще все последние посты с ссылками на либген и рутрекер - мои.
497 846395
>>23017 (OP)
https://www.youtube.com/watch?v=q_kap_FL9NM

Что скажете за сайты типо Briliant?
498 846403
>>46348

>Человек, который осиливает математику, наоборот начинает любить Россию (как Савватеев например).


Савватеев не осилил математику старше то ли 2-го, то ли 3-го курса, как он сам сказал, это раз. Сейчас он вообще, по сути, экономист, а не математик, это два. И три - в математическом сообществе он, скорее, исключение - обычно математики патриотизмом не отличаются, в лучшем случае нейтрально относятся.
499 846406
>>46403
Потому что у него нет анального вектора, а без него нет ученых, только экономисты типо Карла Маркса.
1B5B363E-5265-48CE-9E3B-81C0E3623C40.jpeg184 Кб, 1280x720
500 846407
У Гарварда Фоксфорда какие политические предпочтения?
501 846446
>>46407
ватник и пидораха, очевидно же
502 846475
Посмотрел учебник по астрономии от профессоров МГУ, теперь мне интересно - а есть ли толковые учебники МГУ по математике и геометрии?

Знаю, что Орлов хороший учебник по Истории, я по нему на сотку сдал, но не знаю как именно гуглить такое.
503 846479
>>46475
Серия ВМК-школе хороша, особенно по геометрии. А так, почти в любом коллективе авторов школьных учебников по математике есть имеющие отношение к МГУ люди.
504 846493
>>46479

>ВМК-школе


Спасибо
505 847414
Спрошу совета мудрых - как довести навыки устного счёта до автоматизма? Постоянно делаю ошибки в вычислениях. +- единица до правильного ответа.

И есть ли какие-нибудь сайты, где можно тренироваться в этом навыке?

Сам я находил только одну книгу, где разбирают аж быстрое вычисление значений синусов и косинусов, но я до такого ещё не дошёл.
506 847637
>>47414
Что за книга?
507 847803
>>47637
Dead Reckoning: Calculating Without Instruments by Doerfler
508 848365
>>23017 (OP)
В старой программе Вербицкого есть упоминание начал квантовой механики от Манина, но я не нашёл. У кого-то есть ссылка?
509 848388
>>48365
Разве это не в книге Манина по линейной алгебре?
image.png2 Кб, 355x55
510 848389
image.png152 Кб, 885x366
511 848391
512 848393
>>48391
Ага, вот, теперь понятно. Спасибо.
513 848505
>>46311
Khan Academy это действительно +-сдать экзамены на 3ку после полугода изучения. Вкатывайся в академ.литературу, ломай себя, даже если тебе 30ть.
image.png95 Кб, 758x577
514 848715
ВНИМАНИЕ!!

Где можно подсмотреть передовые образовательные (учебные) программы по дисциплине мaтeмaтика? Я аспергер и не могу начать учиться, не составив для себя идеальный план, просмотрев все возможные варианты. Жду ваших обсуждений итт. Спасибо за внимание.
515 848809
>>48715
Программа Вербицкого.
516 848865
>>48715
ОП-пост посмотри.
517 848901
>>48865
А есть ли смысл брать эту программу Вербицкого и идти по ней по темам, не следуя никаким учебникам, а просто изучая каждую тему по порядку?
518 848906
>>48901
Имеется в виду - решать листочки? Да, так можно, листочки есть где-то до конца второго курса. Но какое-то количество учебников всё равно придётся открывать.
519 848909
>>48906
Какие листочки? Я имел в виду брать тему, гуглить её, узнавать там про неё, искать по ней задачки, решать их и когда уже более менее освоился переходить дальше. Только вот я щас изучил представленные темы и нету например каких-то условных уравнений высших степеней. Тоесть не вся программа описана
520 848928
>>48909
Имеется в виду обучение по системе Константинова, когда вместо чтения учебника человеку предлагается прорешать хорошо подобранные теоретические задачи ("листочек") и как бы самостоятельно сделать открытие. Считается, что обучение по листкам эффективнее чтения учебников.

Например, вот набор листков школьного уровня:
https://mccme.ru/free-books/yaschenko/v08book-08.pdf
https://mccme.ru/free-books/yaschenko/v08book-09.pdf

Листки для основных тем программы Вербицкого есть на сайте матфака и на старом сайте, https://web.archive.org/web/20191021222521/http://vyshka.math.ru/

Способность решить листочки - хороший индикатор успешности обучения. Скажем, если у человека не получается решить листочек по группам, то понимания групп у человека нет. По-видимому, бесполезно изучать математику, не пытаясь решить задачки.

>уравнений высших степеней


Имеются в виду многочленные уравнение, вроде квадратных? ax^n + ... + c = 0? Такие уравнения для степени выше второй изучаются с помощью теории Галуа. Можно, конечно, выучить формулу Кардано (для 3й) и Феррари (для 4й) степеней наизусть, но логичнее научиться выводить подобные формулы самостоятельно с помощью резольвент, как это показано, например, в книжке Постникова. Для уравнения 5й степени и выше аналогичных формул быть не может, кстати, - запрещает теорема Абеля о неразрешимости.
521 848934
>>48901
Программа Вербита - это частично троллинг. Выше второго курса в ней надо дрочить только если тебя интересуют конкретно те вещи, которыми занимается сам Вербит. А первые два курса, конечно, база.
photo2021-05-1101-36-58.jpg15 Кб, 437x546
522 848944
>>48934

>А первые два курса, конечно, база.


Не два, а три. На третьем курсе К-теория, гомоалгебра, пучки, Риман-Рох и формула индекса. На четвёртом уже более специальные вещи, да.
523 848980
>>48944
Частично согласен, но тут уже найдется достаточное количество математиков, которым вся эта гомоалгебра либо не особо нужна, либо не в эти объемах. Дрочить гомологии-когомологии как базу как-то сомнительно.
524 849109
Если у нас есть тождественное преобразование дроби, которое сужает область видимости, то можно сказать, что это преобразование как бы не совсем чистое? Потому что изначальная дробь, которую мы, следуя идее тождественного преобразования должны были просто переписать в более удобном виде, в итоге потеряла одно значение из допустимых, а значит изменилась, как если бы мы просто неправильно её преобразовали.
А вот если у нас расширяющие тождественное преобразование, тогда мы, упуская дополнительные уточнения допустимых значений переменной, ровно как в прошлом случае не совсем чисто преобразовываем дробь, однако в этом случае, в случае расширения, мы можем исправить ситуацию искуственно добавив ограничения для a, тем самым изначальную дробь передав в точности, без всяких расширений и сужений. Для этого есть какие-то умные слова там? Я прав вообще? Может такие преобразования как на 1 пике делать вообще не следует?
525 850401
Племяннику надо пояснить за отрицательные числа. И вот я взялся за эту лёгкую(ага) работку. И знаете что? Пиздец. Почему минус на минус даёт плюс я сам не понимаю. Объясните мне на хуях, пожалуйста.
526 850588
>>50401
Из-за вращения. Тут надо к комплексным числам переходить.
527 850593
>>50588
А здесь нет рекурсии?
528 850594
>>50588
И получается, что пиздюкам надо сразу и комплексные числа проходить?
529 850596
>>50593
Нет. Умножение комплексных чисел геометрически это растяжения и вращения. В действительным случае вращения вырождаются и можно повернуться лишь на 180 градусов и обратно.

>>50594
Так тебе или пиздюку нужно объяснять?
Можешь ещё ролик Трушина глянуть, там без комплексных чисел.
https://youtu.be/JGxEI1hLLuQ
530 850603
>>50596

> тебе или пиздюку


Мне за пиздюков сердце болит. Учат догмам.
531 850608
>>50603
https://youtu.be/b3adw5igSzI
Вот ролик, где геометрически вращения объясняют.
SmartSelect20210520-151551Samsung Notes.jpg122 Кб, 676x882
532 850936
>>50608
Непонятно. Я сам себе придумал через вращение. Ось х - первое слагаемое, ось у - второе. Ось игрек движется вдоль оси икс. Перемещается нулём к первому слагаемому. Плюс - поворот на 90 градусов игрека направо, минус - налево. Результат считается по оси икс (к нему же прибавляется). И когда, к примеру, - 1 - (-3) , то перемещение ноля игрек-оси к единице икса и поворот оси игрек (своей стрелкой) с влево приводит к наложению (-3) на (+2), а следовательно сумма равна двум. Тут ещё можно видеть отличие a+(-b) и a-(+b). Я молодец?
533 854869
>>49109
бамп
534 857727
Если у нас есть тождественное преобразование дроби, которое сужает область видимости, то можно сказать, что это преобразование как бы не совсем чистое? Потому что изначальная дробь, которую мы, следуя идее тождественного преобразования должны были просто переписать в более удобном виде, в итоге потеряла одно значение из допустимых, а значит изменилась, как если бы мы просто неправильно её преобразовали.
А вот если у нас расширяющие тождественное преобразование, тогда мы, упуская дополнительные уточнения допустимых значений переменной, ровно как в прошлом случае не совсем чисто преобразовываем дробь, однако в этом случае, в случае расширения, мы можем исправить ситуацию искуственно добавив ограничения для a, тем самым изначальную дробь передав в точности, без всяких расширений и сужений. Для этого есть какие-то умные слова там? Я прав вообще? Может такие преобразования как на 1 пике делать вообще не следует?
535 862229
>>57727
Равносильные и неравносильные преобразования.
Тред утонул или удален.
Это копия, сохраненная 7 июня 2021 года.

Скачать тред: только с превью, с превью и прикрепленными файлами.
Второй вариант может долго скачиваться. Файлы будут только в живых или недавно утонувших тредах. Подробнее

Если вам полезен архив М.Двача, пожертвуйте на оплату сервера.
« /un/В начало тредаВеб-версияНастройки
/a//b//mu//s//vg/Все доски